Download as pdf or txt
Download as pdf or txt
You are on page 1of 109

MANAGEMENT SERVICES (MS)

FIRST PREBOARD EXAMINATION


MAY 2023 LECPA

Direction: Choose the letter corresponding to the best answer for each of the questions provided below.
This examination consists of 70 items and the exam is good for three (3) hours.

1. Which of the following statements about management or financial accounting is false?


a. Financial accounting must follow GAAP
b. Management accounting is not subject to regulatory reporting standards
c. Both management and financial accounting are subject to mandatory recordkeeping
requirements
d. Management accounting should be flexible

2. Manufacturing overhead includes


a. All direct material, direct labor and administrative costs
b. All manufacturing costs except direct labor
c. All manufacturing costs except direct labor and direct materials
d. All selling and administrative costs

3. Which of the following is not a period cost?


a. Monthly depreciation of the equipment in a fitness room used by factory workers
b. Salary of a billing clerk
c. Insurance on a company showroom
d. Cost of a seminar concerning tax updates that was attended by the company's controller

4. All of the following are examples of product costs except


a. Depreciation on the company's retail outlets
b. Salary of the plant manager
c. Insurance on the factory equipment
d. Rental costs of the factory facility

5. The annual insurance premium for the factory building would be a


a. Fixed cost, period cost, and indirect cost with regard to units of product
b. Fixed cost, product cost, and direct cost with regard to units of product
c. Variable cost, product cost, direct cost with regard to units of product
d. Fixed cost, product cost, indirect cost with regard to units of product

6. An example of a fixed cost that would be considered a direct cost is


a. A cost accountant's salary when the cost object is a unit of product
b. The rental cost of a warehouse to store finished goods when the cost object is the Purchasing
Department
c. A production supervisor's salary when the cost objective is the Production Department
d. Board of Directors' fees when the cost object is the Marketing Department

7. Which of the following will decrease the break-even point?


Decrease in Increase in direct Increase in
fixed cost labor cost selling price
a. Yes Yes Yes
b. Yes No Yes
c. Yes No No
d. No Yes No

8. Prime cost and conversion cost share what common element of total cost?
a. Direct labor
b. Variable overhead
c. Fixed overhead
d. Direct materials
Use the following information for the next three (3) questions:

A partial listing of costs incurred at Pinnacle Corporation during September appears below:
Direct materials ......................................................... P113,000
Utilities, factory ......................................................... P5,000
Administrative salaries .............................................. P81,000
Indirect labor ............................................................. P25,000
Sales commissions..................................................... P48,000
Depreciation of production equipment .................... P20,000
Depreciation of administrative equipment ............... P30,000
Direct labor................................................................ P129,000
Advertising ................................................................ P135,000

9. The total of the manufacturing overhead costs listed above for September is:
a. P586,000
b. P50,000
c. P292,000
d. P30,000

10. The total of the product costs listed above for September is:
a. P292,000
b. P294,000
c. P50,000
d. P586,000

11. The total of the period costs listed above for September is:
a. P294,000
b. P344,000
c. P292,000
d. P50,000

Use the following information for the next three (3) questions:

The following information applies to CPA Company for the current period:

Inventories: March 1 March 31


Raw material P18,000 P15,000
Work in process 9,000 6,000
Finished goods 27,000 36,000

Additional information for March:


Raw material purchased P42,000
Direct labor payroll 30,000
Direct labor rate per hour 7.50
Overhead rate per direct labor hour 10.00

12. The prime cost incurred was


a. P75,000
b. P69,000
c. P45,000
d. P39,000

13. The conversion cost incurred was


a. P30,000
b. P40,000
c. P70,000
d. P72,000

14. The Cost of Goods Manufactured was


a. P118,000
b. P115,000
c. P112,000
d. P109,000
Use the following information for the next three (3) questions:

Cobra Corporation purchased a machine 5 years ago for P527,000 when it launched product M08.
Unfortunately, this machine has broken down and cannot be repaired. The machine could be replaced
by a new model 310 machine costing P545,000 or by a new model 240 machine costing P450,000.

Management has decided to buy the model 240 machine. It has less capacity than the model 310
machine, but its capacity is sufficient to continue making product M08.

Management also considered, but rejected, the alternative of dropping product M08 and not replacing
the old machine. If that were done, the P450,000 invested in the new machine could instead have been
invested in a project that would have returned a total of P532,000.
15. In making the decision to buy the model 240 machine rather than the model 310 machine, the
differential cost was:
a. P95,000
b. P5,000
c. P77,000
d. P18,000

16. In making the decision to buy the model 240 machine rather than the model 310 machine, the sunk
cost was:
a. P545,000
b. P450,000
c. P527,000
d. P532,000

17. In making the decision to invest in the model 240 machine, the opportunity cost was:
a. P545,000
b. P450,000
c. P532,000
d. P527,000

Use the following information for the next four (4) questions:

Andrew is an engineer who has designed a telecommunications device. He is convinced that there is a
big potential market for the device. Accordingly, he has decided to quit his present job and start a
company to manufacture and market the device.
18. The salary that Andrew earns at his present employ is
a. A variable cost
b. A fixed cost
c. A product cost
d. An opportunity cost

19. Andrew purchased a machine two years ago to make experimental boards. The machine will be used
to manufacture the new board. The cost of this machine is
a. An opportunity cost
b. A sunk cost
c. A differential cost
d. A period cost

20. The cost of the raw materials that will be used in manufacturing the computer board is
a. A sunk cost
b. A fixed cost
c. A period cost
d. A variable cost

21. Rent on the administrative office space is


a. A variable cost
b. An opportunity cost
c. A period cost
d. A product cost
Use the following information for the next four (4) questions:

CPA Corporation reported the following data for the month of April:
Inventories: Beginning Ending
Raw materials ....................... P27,000 P20,000
Work in process .................... P10,000 P24,000
Finished goods ...................... P38,000 P28,000

Additional information:
Sales ................................................................. P230,000
Raw materials purchases ................................. P76,000
Direct labor cost............................................... P30,000
Manufacturing overhead cost ......................... P61,000
Selling expense ................................................ P22,000
Administrative expense ................................... P26,000

22. The total manufacturing cost for April was:


a. P61,000
b. P167,000
c. P91,000
d. P174,000

23. The cost of goods manufactured for April was:


a. P160,000
b. P174,000
c. P167,000
d. P188,000

24. The cost of goods sold for April was:


a. P240,000
b. P170,000
c. P150,000
d. P113,000

25. The net operating income for April was:


a. P60,000
b. P15,000
c. P12,000
d. P91,000

26. The coefficient of determination indicates


a. Causal relationships among costs and other factors
b. The percentage of explained variance in the dependent variable
c. The linear relationship between two variables
d. Whether several variables fluctuate

27. The coefficient of correlation that indicates the weakest linear association between two variables is:
a. -0.73
b. -0.11
c. 0.12
d. 0.35

28. If both the fixed and variable expenses associated with a product decrease, what will be the effect on
the contribution margin ratio and the break-even point, respectively?
Contribution margin ratio Break-even point
a. Decrease Increase
b. Increase Decrease
c. Decrease Decrease
d. Increase Increase
Use the following information for the next three (3) questions:

A cement manufacturer has supplied the following data:


Tons of cement produced and sold .............................. 680,000
Sales revenue ................................................................ P2,788,000
Variable manufacturing expense .................................. P1,156,000
Fixed manufacturing expense....................................... P760,000
Variable selling and administrative expense ................ P272,000
Fixed selling and administrative expense ..................... P294,000
Net operating income ................................................... P306,000

29. What is the company's unit contribution margin?


a. P0.45
b. P2.10
c. P2.00
d. P4.10

30. The company's contribution margin ratio is closest to:


a. 39.0%
b. 51.2%
c. 11.0%
d. 48.8%

31. If the company increases its unit sales volume by 4% without increasing its fixed expenses, then total
net operating income should be closest to:
a. P12,240
b. P318,240
c. P360,400
d. P311,973

Use the following information for the next two (2) questions:
The following information relates to Francisca Company:

Degree of operating leverage ............ 2.5


Profit margin percentage ................... 24%
Margin of safety percentage ............. 40%
Contribution margin ratio .................. 60%

32. If Francisca's sales increase by 20%, by what percentage will its net operating income increase?
a. 4.8%
b. 8%
c. 12%
d. 50%

33. Francisca wants to give its sales staff a P60,000 increase in salary but still wants to make the same net
operating income. If Francisca gives this increase, by how much would sales at Francisca have to
increase in order for the company to maintain its current net operating income level?
a. P60,000
b. P100,000
c. P150,000
d. P250,000
Use the following information for the next four (4) questions:

Next year, ABC Company expects to sell 32,000 shirts. ABC is budgeting the following operating results
for next year:
Sales ................................................... P800,000
Variable expenses .............................. 288,000
Contribution margin .......................... 512,000
Fixed expenses ................................... 192,000
Net operating income ........................ P320,000

34. What is the margin of safety for next year?


a. P480,000
b. P500,000
c. P512,000
d. P608,000

35. What is the degree of operating leverage for next year?


a. 1.50
b. 1.56
c. 1.60
d. 2.50

36. How many shirts would have to sell next year in order to generate P480,000 of net operating income?
a. 38,400
b. 48,000
c. 60,000
d. 42,000

37. If ABC is considering increasing its advertising by P48,000 next year. By how much would sales have to
increase in order to still generate a P320,000 net operating income?
a. P48,000
b. P75,000
c. P76,800
d. P120,000

38. Absorption costing differs from variable costing in all of the following, except
a. Treatment of fixed manufacturing overhead
b. Treatment of variable production costs
c. Acceptability for external reporting
d. Arrangement of the income statement

39. Under the variable costing method, which of the following is always expensed in its entirety in the
period in which it is incurred?
a. Fixed manufacturing overhead cost
b. Fixed selling and administrative expense
c. Variable selling and administrative expense
d. All of the above

40. When production exceeds sales, net operating income reported under variable costing generally will
be:
a. Greater than net operating income reported under absorption costing
b. Less than net operating income reported under absorption costing
c. Equal to net operating income reported under absorption costing
d. Higher or lower because no generalization can be made
Use the following information for the next three (3) questions:

During its first year of operations, BSA Manufacturing Company incurred the following costs to produce
8,000 units of its product:
Direct materials ............................................... P7 per unit
Direct labor...................................................... P3 per unit
Variable manufacturing overhead .................. P18 per unit
Fixed manufacturing overhead ....................... P450,000 in total

The company also incurred the following costs in the sale of 7,500 units of product during its first year:
Variable selling and administrative ................. P2 per unit
Fixed selling and administrative ...................... P60,000 in total

Assume that direct labor is a variable cost.

41. What is the total cost that would be assigned to finished goods inventory at the end of the first year of
operations under the absorption costing method?
a. P15,000
b. P42,125
c. P44,000
d. P47,125

42. What is the total cost that would be assigned to finished goods inventory at the end of the first year of
operations under the variable costing method?
a. P15,000
b. P42,125
c. P44,000
d. P14,000

43. If the absorption costing net operating income for this first year is P118,125, what would its variable
costing net operating income be for this first year?
a. P86,000
b. P90,000
c. P104,125
d. P146,250

44. Poorly trained workers could have an unfavorable effect on which of the following variances?

Labor Rate Variance Materials Quantity Variance


a. Yes Yes
b. Yes No
c. No Yes
d. No No

45. ABC Corporation is developing standards for its products. One product requires an input that is
purchased for P82.00 per kilogram from the supplier. By paying cash, the company gets a discount of
2% off this purchase price. Shipping costs from the supplier's warehouse amount to P6.55 per kilogram.
Receiving costs are P0.47 per kilogram. The standard price per kilogram of this input should be:
a. P76.62
b. P87.38
c. P90.66
d. P82.00
Use the following information for the next six (6) questions:

Tristan Company produces a single product. The standard cost card for the product follows:
Direct materials (4 meters @ P5 per meter) ...................................... P20
Direct labor (1.5 hours @ P10 per hour) ............................................ P15
Variable manufacturing overhead (1.5 hrs. @ P4/hour) .................... P6

During a recent period the company produced 1,200 units of product. Various costs associated with
the production of these units are given below:

Direct materials purchased (6,000 meters) .................. P28,500


Direct materials used in production ............................. 5,000 meters
Direct labor cost incurred (2,100 hours) ...................... P17,850
Variable manufacturing overhead cost incurred .......... P10,080

The company records all variances at the earliest possible point in time. Variable manufacturing
overhead costs are applied to products on the basis of direct labor hours.
46. The materials price variance for the period is:
a. P1,250 F
b. P1,500 F
c. P1,250 U
d. P1,500 U

47. The materials quantity variance for the period is:


a. P950 U
b. P5,000 F
c. P1,000 U
d. P6,000 F

48. The labor rate variance for the period is:


a. P3,150 U
b. P2,700 F
c. P2,700 U
d. P3,150 F

49. The labor efficiency variance for the period is:


a. P3,000 U
b. P2,550 U
c. P2,550 F
d. P3,000 F

50. The variable overhead spending variance for the period is:
a. P1,680 F
b. P1,440 U
c. P1,440 F
d. P1,680 U

51. The variable overhead efficiency variance for the period is:
a. P1,200 U
b. P1,440 U
c. P1,200 F
d. P1,440 F
52. ABC Company's direct labor costs for the month of January were as follows:

Actual total direct labor-hours ........................ 20,000


Standard total direct labor-hours .................... 21,000
Direct labor rate variance—unfavorable ......... P3,000
Total direct labor cost...................................... P126,000

What was the direct labor efficiency variance?


a. P6,000 favorable
b. P6,150 favorable
c. P6,300 favorable
d. P6,450 favorable

53. The term "opportunity cost" is best defined as


a. The amount of money paid for an item
b. The amount of money paid for an item, taking possible discounts into account
c. The benefit associated with a rejected alternative when making a choice
d. An irrelevant decision factor

54. A special order generally should be accepted if


a. The revenue exceeds allocated fixed costs, regardless of the variable costs associated with the
order
b. Excess capacity exists and the revenue exceeds all variable costs associated with the order
c. Excess capacity exists and the revenue exceeds allocated fixed costs
d. The revenue exceeds total costs, regardless of available capacity

55. Which of the following standard cost variances would usually be least controllable by a production
supervisor?
a. Fixed overhead volume variance
b. Variable overhead efficiency variance
c. Direct labor efficiency variance
d. Materials usage (quantity) variance

Use the following information for the next two (2) questions:

GHI Company has the capacity to manufacture and sell 5,000 tablets each year but is currently only
manufacturing and selling 4,800. The following per unit numbers relate to annual operations at 4,800
units:
Per tablet
Selling price ..................................................... P600
Manufacturing costs:
Variable ........................................................ P130
Fixed............................................................. P270
Selling and administrative costs:
Variable ........................................................ P20
Fixed............................................................. P40

JKL Corp. is interested in purchasing GHI’s excess capacity of 200 units but only if they can get the
tablets for P350 each. This special order would not affect regular sales or the cost structure above.
56. If the special order from JKL is accepted, GHI’s profits for the year will:
a. Increase by P40,000
b. Decrease by P10,000
c. Decrease by P22,000
d. Decrease by P28,000

57. Assume that GHI is manufacturing and selling at capacity (5,000 units). Any special order will mean a
loss of regular sales. Under these conditions if the special order from JKL is accepted, GHI’s profits for
the year will decrease by:
a. P20,000
b. P22,000
c. P28,000
d. P50,000
58. CPA Company produces 1,000 units of a part per year which are used in the assembly of one of its
products. The unit cost of producing these parts is:

Variable manufacturing cost .............. P15


Fixed manufacturing cost ................... 12
Total manufacturing cost.................... P27

The part can be purchased from an outside supplier at P20 per unit. If the part is purchased from the
outside supplier, two thirds of the total fixed costs incurred in producing the part can be eliminated.
The annual increase or decrease on the company's operating incomes as a result of buying the part
from the outside supplier would be:
a. P3,000 increase
b. 1,000 decrease
c. P7,000 increase
d. P5,000 decrease

59. ABM Company produces three products, with costs and selling prices as follows:

Product A Product B Product C


Selling price per unit .................... P30 100% P20 100% P15 100%
Variable costs per unit ................. 18 60% 15 75% 6 40%
Contribution margin per unit ...... P12 40% P5 25% P9 60%

A particular machine is a bottleneck. On that machine, 3 machine hours are required to produce each
unit of Product A, 1 hour is required to produce each unit of Product B, and 2 hours are required to
produce each unit of Product C. In which order should it produce its products?
a. C, A, B
b. A, C, B
c. B, C, A
d. The order of production doesn't matter

Use the following information for the next two (2) questions:

Hi-Tech manufactures two products: Regular and Super. The results of operations for 2025 follow.

Regular Super Total


Units 10,000 3,700 13,700
Sales P P240,000 P740,000 P980,000
Less: Cost of goods sold 180,000 481,000 661,000
Gross margin P60,000 P259,000 P319,000
Less: Selling expenses 60,000 134,000 194,000
Operating income 0 P125,000 P125,000

Fixed manufacturing costs included in cost of goods sold amount to P3 per unit for Regular and P20
per unit for Super. Variable selling expenses are P4 per unit for Regular and P20 per unit for Super;
remaining selling amounts are fixed.

60. Hi-Tech wants to drop the Regular product line. If the line is dropped, company-wide fixed
manufacturing costs would fall by 10% because there is no alternative use of the facilities. What would
be the impact on operating income if Regular is discontinued?
a. P0
b. P10,400 increase
c. P20,000 increase
d. P39,600 decrease

61. Disregard the information in the previous question. If Hi-Tech eliminates Regular and uses the
available capacity to produce and sell an additional 1,500 units of Super, what would be the impact on
operating income?
a. P28,000 increase
b. P45,000 increase
c. P55,000 increase
d. P85,000 increase
62. XYZ Corporation has P200,000 of joint processing costs and is studying whether to process J and K
beyond the split-off point. Information about J and K follows.

Product J Product K
Tons produced 25,000 15,000
Separable variable processing costs beyond split-off P64,000 P100,000
Selling price per ton at split-off 15 52
Selling price per ton after additional processing 21 58

If XYZ desires to maximize total company income, what should the firm do with regard to Products J
and K?
Product J Product K
a. Sell at split-off Sell at split-off
b. Sell at split-off Process beyond split-off
c. Process beyond split-off Sell at split-off
d. Process beyond split-off Process beyond split-off

63. Which of the following equations can be used to budget purchases?

(BI = beginning inventory, EI = ending inventory desired, CGS = budgeted cost of goods sold, P =
budgeted purchases)
a. P = CGS + BI – EI
b. P = CGS + BI
c. P = CGS + EI + BI
d. P = CGS + EI – BI

64. Which of the following represents a proper sequencing in which the budgets below are prepared?
a. Direct Material Purchases, Cash, Sales
b. Production, Sales, Income Statement
c. Sales, Balance Sheet, Direct Labor
d. Sales, Production, Manufacturing Overhead

Use the following information for the next two (2) questions:

XYZ manufactures a product requiring 0.5 ounces of platinum per unit. The cost of platinum is
approximately P360 per ounce; the company maintains an ending platinum inventory equal to 10% of
the following month's production usage. The following data were taken from the most recent
quarterly production budget:
July August September
Planned production in units 1,000 1,100 980

65. The cost of platinum to be purchased to support August production is:


a. P195,840
b. P198,000
c. P200,160
d. P391,680

66. If it takes two direct labor hours to produce each unit and the cost per labor hour is P15, direct labor
cost for August would be budgeted at:
a. P16,500
b. P31,200
c. P33,000
d. P34,800
Use the following information for the next two (2) questions:

BSA Company's budgeted income statement reflects the following amounts:

Sales Purchases Expenses


January P120,000 P78,000 P24,000
February 110,000 66,000 24,200
March 125,000 81,250 27,000
April 130,000 84,500 28,600

Sales are collected 50% in the month of sale, 30% in the month following sale, and 19% in the second
month following sale. 1% of sales is uncollectible and expensed at the end of the year.

BSA pays for all purchases in the month following purchase and takes advantage of a 3% discount. The
following balances are as of January 1:

Cash P88,000
Accounts receivable* 58,000
Accounts payable 72,000

*Of this balance, P35,000 will be collected in January and the remaining amount will be collected in
February.

The monthly expense figures include P5,000 of depreciation. The expenses are paid in the month
incurred.
67. The budgeted cash receipts in February are
a. P91,000
b. P95,000
c. P113,640
d. P114,000

68. The budgeted cash payments in February are


a. P75,660
b. P94,860
c. P97,200
d. P99,860

Use the following information for the next two (2) questions:

ABC, Inc. analyzes manufacturing overhead in the production of its only one product. The following set of
information applies to the month of December:

Budgeted Actual
Units produced 20,000 19,000
Variable manufacturing overhead P 4/DLH P8,200
Fixed manufacturing overhead P20/DLH P44,000
Direct labor hours 6 minutes/unit 2,100 hours

69. What is the fixed overhead spending variance?


a. P2,000 Favorable
b. P4,000 Favorable
c. P4,000 Unfavorable
d. P2,000 Unfavorable

70. What is the volume variance?


a. P2,000 Favorable
b. P2,000 Unfavorable
c. P4,000 Favorable
d. P4,000 Unfavorable

- - - END - - -
AUDITING
FIRST PREBOARD EXAMINATION
MAY 2023 LECPA

Direction: Choose the letter corresponding to the best answer for each of the questions provided below. This
examination consists of 70 items and the exam is good for three (3) hours.

THEORY

1. Which of the following best describes the objective of an audit of financial statements?
a. To express an assurance about the management’s efficiency or effectiveness in conducting the
operations of entity
b. To express an opinion whether the financial statements are prepared, in all material respect,
in accordance with an identified financial reporting framework
c. To express an opinion whether the financial statements are prepared in accordance with
prescribed criteria
d. To express an assurance as to the future viability of the entity whose financial statements are
being audited

2. Which of the following can be controlled by the auditor?


a. Inherent risk
b. Control risk
c. Detection risk
d. Both detection and control risk

3. Which of the following statements regarding pre-engagement activities is false?


a. An engagement letter represents a written contract between the auditors and the client
b. Successor auditors are required by professional standards to make certain inquiries of the
predecessors before accepting a new audit client
c. Two of the above statements are false
d. CPAs cannot ethically refuse to provide auditing services to any prospective client which the
CPAs are competent to audit, if the prospective client is willing to pay the CPAs’ standard
audit fees

4. A CPA is conducting the first examination of a company's financial statements. The CPA hopes to
reduce the audit work by consulting with the predecessor auditor and reviewing the predecessor's
working papers. This practice is
a. Acceptable if the CPA refers in the audit report to reliance upon the predecessor auditor's
work
b. Required if the CPA is to render an unqualified opinion
c. Acceptable if the client and the predecessor auditor agree to it
d. Unacceptable because the CPA should bring an independent viewpoint to a new engagement

5. There is an inverse relationship between the effectiveness of an entity's internal control structure
and the
a. Reliability of financial statements
b. Degree of staff supervision required in the performance of an audit
c. Extent of detailed substantive procedures required
d. Fairness of management assertions in the financial statements

6. When perpetual inventory records are maintained in quantities and in pesos, and internal accounting
control over inventory is weak, the auditor would probably
a. Increase the extent of tests for unrecorded liabilities at the end of the year
b. Have to disclaim an opinion on the income statement for that year
c. Want the client to schedule the physical inventory count at the end of the year
d. Insist that the client perform physical counts of inventory items several times during the year

7. Which of the following procedures ordinarily performed during an audit are also performed in
review?
a. Assessment of accounting and internal control systems
b. Test of controls
c. Inquiry and analytical procedures
d. Tests of records and of responses to inquiries
8. After obtaining an understanding of an entity’s internal controls, an auditor may assess control risk at
the maximum for some assertions because the auditor
a. Determines that internal control is not well-documented
b. Performs tests of controls to restrict detection of risk to an acceptable level
c. Believes internal control activities are unlikely to be effective
d. Identifies control activities that are likely to prevent material misstatements

9. Which of the following is appropriate about risk assessment?


a. Detection risk is eliminated if an auditor were to examine 100% of the account balance or
class of transactions
b. Audit risk may be more appropriately determined by assessing inherent and control risk
separately
c. There is an inverse relationship between detection risk and the combined level of inherent
and control risk
d. The assessed level of inherent and control risk can be sufficiently low, thus resulting to
eliminating the need for substantive tests

10. Which element of quality control is addressed by the establishment of policies and procedures
designed to provide the firm with reasonable assurance that it has sufficient personnel with the
competence, capabilities and commitment to ethical principles?
a. Human resources
b. Monitoring
c. Engagement performance
d. Leadership responsibilities for quality within the firm

11. Which of the following is an example of fraudulent financial reporting?


a. An employee steals inventory, and the “shrinkage” is recorded in cost of goods sold
b. The treasurer diverts customer payments to his personal due, concealing his actions by
debiting an expense account, thus overstating expenses
c. An employee steals small tools from the company and neglects to return them; the cost is
reported as a miscellaneous operating expense
d. Company management changes inventory count tags and overstates ending inventory, while
understating cost of goods sold

12. Not all engagements performed by professional accountants are assurance engagements. Other
engagements frequently performed by professional accountants that are non-assurance
engagements include the following, except
a. Agreed-upon procedures
b. Review of financial statements
c. Management consulting
d. Compilation of financial or other information

13. According to Philippine Standard on Auditing, the procedures employed in doing compilation are
a. Designed to enable the accountant to express a limited assurance
b. Designed to enable the accountant to express a negative assurance
c. Less extensive than review procedures but more extensive than agreed-upon procedures
d. Not designed to enable the accountant to express any form of assurance

14. The accounts payable department usually has the responsibility for verifying the propriety of
acquisitions by comparing the details on the
a. Vendor’s invoice and the receiving report
b. Purchase requisition, purchase order and receiving report
c. Purchase order, receiving report, and vendor’s invoice
d. Vendor’s invoice and the purchase requisition

15. As the acceptable level of detection risk decreases, the assurance directly provided from
a. Tests of controls should increase
b. Tests of controls should decrease
c. Substantive tests should increase
d. Substantive tests should decrease
16. The four major steps in conducting an audit are
i. Testing internal controls
ii Audit report
iii. Planning
iv. Testing transactions and balances
The proper sequence in applying the above steps is
a. iii, i, iv, ii
b. iii, iv, I, ii
c. ii, iii, iv, i
d. I, iv, iii, ii

17. A practitioner’s report on agreed-upon procedures that is in the form of procedures and findings
should contain
a. Negative assurance that the procedures did not necessarily disclose all reportable conditions
b. A statement of restrictions on the use of the report
c. A disclaimer of opinion on the entity’s financial statements
d. An acknowledgment of the practitioner’s responsibility for the sufficiency of the procedures

18. Inherent risk and control risk differ from detection risk in that inherent risk and control risk are
a. Elements of audit risk while detection risk is not
b. Changed at the auditor’s discretion while detection risk is not
c. Functions of the client and its environment while detection risk is not
d. Considered at the individual account-balance level while detection risk is not

19. One of the major parts of audit stages is pre-engagement activities. Which of the following is not
involved during this phase?
a. Deciding whether to accept or continue this client
b. Selecting staff for the engagement
c. Obtaining information about client’s legal obligations
d. Obtaining an engagement letter

20. Which one of these is not a type of evidence that would be used for both obtaining an understanding
of the control structure and testing the controls?
a. Reperformance
b. Inquiries
c. Inspection
d. Observation

21. To test for recorded sales for which there were no actual shipments, the auditor traces from the
a. Sales journal to the accounts receivable subsidiary ledger
b. Bill of lading to the supporting customer order and sales order
c. Bill of lading to the sales journal
d. Sales journal to the bill of lading

22. An effective procedure to test for unbilled shipment is to trace from the
a. Sales journal to the accounts receivable ledger
b. Sales journal to the general ledger sales account
c. Sales journal to the shipping documents
d. Shipping documents to the sales journal

23. In the purchasing/disbursement cycle, of the order of activities listed, which is in the best order of a
typical sequence of activities?
a. Ordering goods, receiving vendor’s invoice, receiving goods, payment activity
b. Selecting authorized vendor, receiving vendor’s invoice, receiving goods, payment activity
c. Ordering goods, receiving vendor’s invoice, selecting authorized vendor, payment activity
d. Ordering goods, receiving goods, receiving vendor’s invoice, cash payments
24. During audit planning, the auditor uses analytical procedures primarily to
a. Determine the correspondence of the company’s financial statements to the valuation and
accuracy audit objectives
b. Understand the client’s business and industry and to indicate possible misstatements
c. Identify weaknesses in internal control
d. Determine if the company’s financial statements appear reasonable and are free of material
misstatements

25. When inherent risk is high, there will need to be


A lower More evidence
assessment of audit risk accumulated by the auditor
a. Yes Yes
b. Yes No
c. No Yes
d. No No

26. If an auditor establishes a relatively high level for materiality, then the auditor will:
a. Accumulate approximately the same evidence as would be the case were materiality lower
b. Accumulate an undetermined amount of evidence
c. Accumulate more evidence than if a lower level had been set
d. Accumulate less evidence than if a lower level had been set

27. Which of the following statements is true?


a. If control risk is assessed at maximum, the timing of related substantive tests should be
changed from year-end to an interim date
b. If control risk is assessed at maximum, the extent of related substantive tests should be
changed from a larger to a smaller sample
c. If control risk is assessed at maximum, the nature of related substantive tests should be
changed from more to less effective
d. If control risk is assessed at maximum, the nature of related substantive tests should be
changed from less to more effective

28. Inherent risk is _______ related to detection risk and _______ related to the amount of audit
evidence
a. Directly, inversely
b. Directly, directly
c. Inversely, directly
d. Inversely, inversely

29. Which of the following statements is correct with respect to separation of duties?
a. It is permissible to allow an employee to open cash receipts and record those receipts
b. Employees who authorize transactions should have recording responsibility for these
transactions
c. Employees should not have temporary and permanent custody of assets
d. Employees who authorize transactions should not have custody of related assets

30. Which of the following topics is not normally included in an engagement letter?
a. Limitations on the scope of the engagement
b. A description of responsibility for the detection of fraud
c. The auditors' preliminary assessment of internal control
d. The auditors' estimate of the fee for the engagement
31. Which of the following factors most likely would cause a CPA to not accept a new audit
engagement?
a. The prospective client has fired its prior auditor
b. The prospective client is unwilling to make financial records available to the CPA
c. The CPA lacks a thorough understanding of the prospective client's operations and industry
d. The CPA is unable to review the predecessor auditor's working papers
32. Which is most likely when the assessed level of control risk increases?
a. Change from performing substantive procedures at year-end to an interim date
b. Use larger sample sizes for substantive procedures
c. Perform substantive procedures directed inside the entity rather than tests directed toward
parties outside the entity
d. Use the maximum number of dual purpose tests
33. Which of the following would be least likely to be regarded as a test of a control?
a. Tests of signatures on purchase orders
b. Tests of the additions to property by physical inspection
c. Comparisons of the signatures on cancelled checks to the authorized check signer list
d. Observation of factory workers’ performance of duties
34. Which of the following statements concerning materiality thresholds is incorrect?
a. Aggregate materiality thresholds are a function of the auditor's preliminary judgments
concerning audit risk
b. Materiality thresholds may change between the planning and review stages of the audit.
These changes may be due to quantitative and/or qualitative factors
c. The smallest aggregate level of errors or fraud that could be considered material to any one
of the financial statements is referred to as a "materiality threshold"
d. In general, the more misstatements the auditor expects, the higher should be the aggregate
materiality threshold
35. Which of the following controls most likely would be effective in offsetting the tendency of sales
personnel to maximize sales volume at the expense of high bad debt write-offs?
a. Employees involved in the credit-granting function are separated from the sales function
b. Subsidiary accounts receivable records are reconciled to the control account by an employee
independent of the authorization of credit
c. Employees responsible for authorizing sales and bad debt write-offs denied access to cash
d. Shipping documents and sales invoices are matched by an employee who does not have
authority to write off bad debts
36. Among the possible reasons why an auditor will discontinue servicing an audit client is
a. Too many errors have to be adjusted to make the financial statements conform with GAAPs
b. The auditor is also rendering at the same time, a management advisory engagement for the
same client, and the audit was the engagement contracted first
c. The auditor has to use a specialist in verifying inventory valuation
d. A change in the client management and the auditor is worried about the reputation of the
new management
37. Which of the following is an effective internal accounting control measure that encourages receiving
department personnel to count and inspect all merchandise received?
a. Quantities ordered are excluded from the receiving department copy of the purchase order
b. Internal auditors periodically examine, on a surprise basis, the receiving department copies
of receiving reports
c. Vouchers are prepared by accounts payable department personnel only after they match
item counts on the receiving report with the purchase order
d. Receiving department personnel are expected to match and reconcile the receiving report
with the purchase order
38. How can an auditor test to determine whether Receiving Department procedures are applied
properly?
a. Test a sample of receiving documents
b. Observe receiving procedures on a surprise basis
c. Review procedures manuals
d. Interview Receiving personnel

39. A firm of CPAs may use policies and procedures such as notifying professional personnel as to the
names of audit clients having publicly held securities and confirming periodically with such personnel
that prohibited relations do not exist. This is done to achieve effective quality control in which of the
following areas?
a. Leadership responsibilities for quality within the firm
b. Human resources
c. Ethical requirements
d. Acceptance and continuance of clients

40. Which of the following is not done by an auditor when obtaining an understanding of an entity's
internal controls?
a. Consider factors that affect the risk of material misstatements
b. Design substantive tests
c. Consider the operating effectiveness of the internal controls
d. Identify the types of potential misstatements that can occur
41. An auditor has concluded that a client's internal controls are well designed and functioning as
expected. Under these circumstances the auditor would most likely
a. Not increase the extent of planned substantive tests
b. Perform all tests of controls to the extent outlined in the audit program
c. Cease to perform further substantive tests
d. Increase the extent of planned analytical procedures

42. Effective internal control over unclaimed payroll checks that are kept by the Treasury Department
would include Accounting Department procedures that require
a. Accounting for all unclaimed wages in a current liability account
b. Preparation of a list of unclaimed wages on a periodic basis
c. Effective cancellation and stop payment orders for the related checks
d. Periodic accounting for the actual checks representing unclaimed wages

43. Of the following statements about internal control, which one is not valid?
a. No one person should be responsible for the custodial responsibility and the recording
responsibility for an asset
b. Control activities reasonably ensure that collusion among employees can not occur
c. Transactions must be properly authorized before such transactions are processed
d. Because of the cost/benefit relationship, a client may apply control procedures on a test
basis
44. Which of the following is not a component of audit planning?
a. Making arrangements with the client concerning the timing of audit field work and use of the
client's staff in completing certain phases of the examination
b. Developing audit programs
c. Obtaining an understanding of the business
d. Observing the client's annual physical inventory taking and making test counts of selected
items
45. A secondary objective of the auditor's study and evaluation of internal control is that the study and
evaluation provide
a. A basis for constructive suggestions concerning improvements in internal control
b. An assurance that the records and documents have been maintained in accordance with
existing company policies and procedures
c. A basis for reducing the auditor's assessed level of control risk below the maximum level
d. A basis for determination of the resultant extent of the tests to which auditing procedures are
to be restricted

46. An accountant providing a compilation service must


a. Carefully review the information provided by the client
b. Be independent of the client
c. Provide limited assurance that the information follows GAAP
d. Assure that the work is performed by the person with adequate training and with professional
care

47. Which of the following statements is not true?


a. Inherent risk is the susceptibility of the financial statements to material error, assuming no
internal controls
b. Inherent risk is inversely related to evidence
c. Inherent risk is inversely related to detection risk
d. Inherent risk is the auditor’s assessment of the likelihood that errors exceeding a tolerable
amount exist in a segment before considering the effectiveness of internal controls

48. When the auditor attempts to understand the operation of the accounting system by tracing a few
transactions through the accounting system, the auditor is said to be
a. Tracing
b. Vouching
c. Performing a walkthrough
d. Testing controls
49. On financial statement audits, it is required that the auditors obtain an understanding of internal
control, including:
a. Performing tests of controls for all material controls
b. Its ability to provide reasonable assurance
c. Its operating effectiveness
d. Whether it has been implemented
50. Which of the following would be the best procedure to determine whether purchases were properly
authorized?
a. Discuss authorization procedures with personnel in the controller's and purchasing functions
b. Determine whether a sample of entries in the purchase journal is supported by properly
executed purchase orders
c. Review and evaluate a flowchart of purchasing procedures
d. Vouch payments for selected purchases to supporting receiving reports
51. Which of the following is correct with respect to the use of analytical procedures?
a. Analytical procedures used in the testing phase of the audit are primarily used to direct an
auditor’s attention so that the auditor’s understanding of the business is improved
b. Analytical procedures may be used in evaluating balances in the testing phase
c. Analytical procedures must be used throughout the audit
d. Analytical procedures are performed by studying plausible relationships between financial
and nonfinancial data
52. Which statement is incorrect regarding the extent of tests of controls?
a. If the rate of expected deviation is expected to be too high, the auditor may determine that
tests of controls for a particular assertion may not be effective
b. Because of the inherent consistency of IT processing, the auditor may not need to increase the
extent of testing of an automated control
c. The auditor designs tests of controls to obtain sufficient appropriate audit evidence that the
controls operated effectively throughout the period of reliance
d. The more the auditor relies on the operating effectiveness of controls in the assessment of
risk, the lesser is the extent of the auditor’s tests of controls

53. Which of the following control procedures could prevent or detect errors or frauds arising from
shipments made to unauthorized parties?
a. Prenumber bills of lading and assure that related billings are made on a periodic basis
b. Prepare and periodically update lists of authorized customers
c. Document policies and procedures for scheduling shipments
d. Establish procedures for reviewing and approving prices and sales terms before sale

54. Which of the following is not a likely procedure to support the operating effectiveness of internal
controls?
a. Inquiry of client personnel
b. Reperformance of client procedures
c. Completing an internal control questionnaire
d. Observation of control-related activities

55. In connection with the study of internal control, an auditor encounters the following flowcharting
symbols

The auditor would conclude that


a. A document has been generated by a computer operation
b. A master file has been created by a manual operation
c. A document has been generated by a manual operation
d. A master file has been created by a computer operation
PROBLEM 1

You are auditing the accounts receivable of Pinnacle Company as of December 31, 2025. You found the
following information in the general journal:

Accounts receivable P1,466,720


Less: Allowance for doubtful accounts (46,720)
Net realizable value 1,420,000

The accounts receivable subsidiary ledger had the following details:

Customer Invoice Date (M/D/Y) Amount Balance


A 09/12/2025 P139,200 P139,200

B 12/12/2025 153,600
12/02/2025 99,200 252,800

C 11/17/2025 185,120
10/08/2025 176,000 361,120

D 12/08/2025 160,000
10/25/2025 44,800
08/20/2025 40,000 244,800

E 09/27/2025 96,000 96,000

F 08/20/2025 71,360 71,360

G 12/06/2025 112,000
11/29/2025 169,440 281,440
P1,446,720

Additional information:

a. You discovered based on your review of subsequent events that Customer F recently went bankrupt, thus
suggesting that the amount receivable from the same shall be written off.

b. You also discovered that the invoice dated 12/02/2025 has already been settled by Customer B per OR #
123. This amount, however, has been erroneously posted against Customer C’s subsidiary ledger as
settlement for an invoice dated 11/05/2025 for the same amount.

c. The estimated bad debt rates below are based on the company’s receivable collection experience:

Age of Accounts Uncollectibility %


0-30 days 2%
31-60 days 5%
61-90 days 10%
91-120 days 20%
Over 120 days 50%
Requirements:

56. What is the net adjustment to the subsidiary ledger?


a. P27,840
b. P71,360
c. P99,200
d. P170,560

57. What is the adjusting journal entry to reconcile any difference between the general and subsidiary
ledger, assuming the difference is due to an error in recording sales?
a. Dr. Bad Debt Expense 20,000
Cr. Accounts Receivable 20,000
b. Dr. Sales 20,000
Cr. Accounts Receivable 20,000
c. Dr. Miscellaneous Expense 20,000
Cr. Accounts Receivable 20,000
d. Dr. Accounts Receivable 20,000
Cr. Miscellaneous Income 20,000

58. What is the audited accounts receivable balance (gross) for the year ended December 31, 2025?
a. P1,375,360
b. P1,275,040
c. P1,395,360
d. P1,411,040

59. Assuming that there were no other entries to the allowance for doubtful accounts, what is the
correct bad debt expense for 2025?
a. P92,704
b. P95,680
c. P141,984
d. P144,960

60. What is the audited allowance for bad debts for the year ended December 31, 2025?
a. P117,344
b. P120,320
c. P153,024
d. P156,000
PROBLEM 2

BSA Corporation engaged an independent CPA to perform an audit for the year ended December 31, 2025.
The company uses a periodic inventory system. The CPA did not observe the inventory count on December 31,
2025, as a result, a special examination was made of the inventory records.

The financial statements prepared by the company showed the following unadjusted balances: ending
inventory, P72,000; accounts receivable, P60,000; accounts payable, P30,000; sales, P400,000; net purchases,
P160,000, and net income P51,000.

The following data were found during the audit:


a. Merchandise received on January 2, 2026, costing P800 was recorded on December 31, 2025. An
invoice on hand showed the shipment was made FOB shipping point on December 31, 2025. Because
the merchandise was not on hand on December 31, 2025, it was not included in the inventory.

b. Merchandise that cost P18,000 was excluded from the inventory, and the related sale for P23,000 was
recorded. The goods had been segregated in the warehouse for shipment; there was no contract of
sale but a “tentative order by phone”.

c. Merchandise that cost P10,000 was out on consignment for ABC Company and was excluded from the
ending inventory. The merchandise was recorded as a sale P25,000 when shipped to ABC on December
29, 2025.

d. A sealed packing case containing a product costing P900 was in BSA’s shipping room when the physical
inventory was taken. It was included in the inventory because it was marked “Hold for customer’s
shipping instructions.” Investigation revealed that the customer signed a purchase contract dated
December 18, 2025, but that case was shipped and the customer billed on January 10, 2026. A sale
for P1,500 was recorded on December 31, 2025.

e. A special item, fabricated to order for a customer, was finished and in the shipping room on December
30, 2025, but was actually shipped on December 31 FOB shipping point. The customer has inspected
it and was satisfied. The customer was billed in full on that sale in the amount of P5,000. The item
was included in inventory at cost, P1,000.

f. Merchandise costing P15,600 was received on December 28, 2025. The goods were excluded from
inventory, and a purchase was not recorded. The auditor located the related papers in the hands of
the purchasing; they indicated, “On consignment from XYZ Company”.

g. Merchandise that cost P6,000 was excluded from the ending inventory and not recorded as a sale for
P7,500 on December 31, 2025. The goods had been specifically segregated. According to the terms
of the contract of sale, ownership will not pass until actual delivery.

h. Merchandise that cost P15,000 was included in the ending inventory. The related purchase has not
been recorded. The goods had been shipped by the vendor FOB destination, and the invoice was
received on December 30, 2025. The goods was received on January 5, 2026.

i. Merchandise in transit that cost P7,000 was excluded from inventory because it was not on hand. The
shipment from the vendor was FOB shipping point. The purchase was recorded on December 29, 2025,
when the invoice was received.

j. Merchandise in transit that cost P13,000 was excluded from inventory because it had not arrived.
Although the invoice had arrived, the related purchase was not recorded by December 31, 2025. The
merchandise shipped FOB shipping point by the vendor.

k. Merchandise that cost P8,000 was included in the ending inventory because it was on hand. The
merchandise had been rejected because of incorrect specifications and was being held for return to
the vendor. The merchandise was recorded as a purchase on December 26, 2025.
Requirements: Based on your analysis and the information above, answer the following:
61. The adjusted balance of inventory at year-end is:
a. P101,900
b. P102,000
c. P120,400
d. P102,800

62. The adjusted balance of accounts receivable at year-end is:


a. P10,500
b. P12,000
c. P35,000
d. P37,000

63. The adjusted balance of accounts payable at year-end is:


a. P22,000
b. P30,000
c. P35,000
d. P43,000

64. The adjusted balance of Sales at year-end is:


a. P347,000
b. P350,500
c. P352,000
d. P377,000

65. The adjusted balance of Net Purchases at year-end is:


a. P152,000
b. P165,000
c. P173,000
d. P181,000

66. The adjusted balance of Net Income at year-end is:


a. P27,300
b. P29,000
c. P29,800
d. P35,800
PROBLEM 3

You were assigned to audit Mr. Accounting Company’s property, plant and equipment for the year ended
December 31, 2025. The following information were made available as of the beginning of 2025:

Cost Accumulated Depreciation


Land P5,000,000 -
Building 10,000,000 P3,150,000
Factory Equipment 8,000,000 3,904,000
Automotive Equipment 5,000,000 2,700,000

All the company’s properties were acquired upon the commencement of the operations on January 1, 2022
and remained held until the current year. Depreciation is computed based on the following methods and useful
lives with salvage value assumed to be at 10% of the asset’s cost:

Depreciation Method Useful Life


Building Sum-of-years’-digits 15 years
Factory Equipment Double-declining 10 years
Automotive Equipment Straight-line 5 years

The following transactions occurred during the year:

a. A new factory equipment was acquired on June 1, replacing an old factory equipment originally acquired
at P1,500,000 and was disposed on the same date at P250,000. The new equipment was acquired at
P2,000,000, payable 50% down payment, balance payable in four equal installments every June 1 starting
next year. Freight and unloading costs amounted to P50,000 while installation costs amounted to P70,000.

The company estimates that it will incur significant dismantling cost upon retirement of the same factory
equipment. Future estimated dismantling cost at P227,108. The market rate of interest that reflects all
transactions on this date was at 10%. The relevant present value factors are shown below:

Present value of ordinary annuity at 10% for 4 periods 3.1698


Present value of 1 at 10% for 10 periods 0.3855

b. On August 1, an old automotive equipment was sold for P1,200,000. Its original cost was P1,000,000. On
the same date, the proceeds were used to buy a new automotive equipment.

c. Significant improvements on the ventilating system and electrical wiring system of the building were made
at the beginning of the current year. Total cost incurred were P400,000 for the ventilating system and
P380,000 for electrical wiring system. The improvements are to be depreciated separately using the same
depreciation method and remaining useful life of the building but with no salvage value.
Requirements:

67. What is the net gain/loss on disposal of the factory equipment on June 1 and automotive equipment
on August 1?
a. P299,000 net gain
b. P391,000 net gain
c. P391,000 net loss
d. P462,667 net gain

68. Total depreciation expense in 2025


a. P2,897,933
b. P2,999,600
c. P2,920,433
d. P3,064,600

69. Audited balance of the cost of property, plant, and equipment as of December 31, 2025
a. P24,480,000
b. P28,280,000
c. P28,700,000
d. P29,480,000

70. Audited balance of accumulated depreciation as of December 31, 2025


a. P11,090,933
b. P11,210,933
c. P11,312,600
d. P11,377,600

--- END ---


TAXATION
FIRST PREBOARD EXAMINATION
MAY 2023 LECPA

Direction: Choose the letter corresponding to the best answer for each of the questions provided below.
This examination consists of 70 items and the exam is good for three (3) hours.

1. A citizen of the Philippines who works and whose employment requires him to be physically
present abroad most of the time during the taxable year is
a. Taxable on income within and without the Philippines
b. Taxable on income from within the Philippines
c. Taxable on income from without the Philippines
d. Exempt from income tax

2. Which is not a Constitutional Limitation?


a. Due process of law in taxation
b. Free worship clause
c. Exemption of the government
d. Non-diversification of tax collections

3. Mr. X is a purely self-employed individual and his gross sales/receipts and other non-operating
income do not exceed the VAT threshold. He signified his intention to avail of the 8% income tax
rate on his first quarter return, he will pay:
a. Percentage tax under Section 116 in lieu of income tax
b. Income tax on gross sales/receipts and other non-operating income in excess of P250,000 in
lieu of graduated tax rates and percentage under Section 116
c. Income tax on gross sales/receipts and other non-operating income in excess of P250,000
and the percentage tax under Section 116
d. Income tax on gross sales/receipts excluding non-operating income in excess of P250,000 in
lieu of graduated tax rates and percentage under Section 116

4. Which of the following statements is incorrect?


a. A joint venture for undertaking construction projects is not taxable as a corporation
b. An ordinary partnership in trade is taxable as a corporation
c. A co-ownership where the activities of the co-owners are limited to the preservation of
property and collection of income from the property is not taxable as a corporation
d. A consortium for energy operations pursuant to an operating consortium agreement under
a service contract with the government is taxable as a corporation

5. Which of the following is taxable?


a. Retirement pay received from a benefit plan registered with the BIR where at the time the
employee retired he was 55 years of age, retiring from employment for the first time in his
life, and was employed with the employer from whom retiring for 12 years prior to retirement
b. Social security benefit received by a balikbayan from employer abroad at the age of 35
c. Separation pay received by a 50-year old employee due to voluntary resignation
d. All of the above are not taxable

6. Which of the following situs of taxation is incorrect?


a. Gain from sale of tangible personal property – location of property
b. Business income – place of business
c. Gain from sale of shares of stocks – nationality of the issuing corporation
d. Gain from sale of real property – place of sale
7. Statement I: Salaries received by a partner from an ordinary partnership is not considered gross
compensation income but as part of his share in the distributable net income after tax of the
partnership.

Statement II: Salaries received by a partner from a general professional partnership is considered
gross compensation income.

Statement III: Taxable partnerships are required to file cumulative quarterly declarations and a
final income tax return because they are taxed as corporations.

Statement IV: The distributable net income of a taxable partnership shall not include income which
are subjected to final tax as well as those that are exempted from income tax
a. Only one (1) of the above statements is correct
b. Only two (2) of the above statements are correct
c. Three (3) of the above statements are correct
d. All of the above statements are correct

8. Which of the following statements is incorrect?


a. For purposes of computing the distributive share of the partners, the net income of the
general professional partnership shall be computed in the same manner as a corporation
b. Each partner shall report as gross income his distributive share, actually or constructively
received, in the net income of the general professional partnership
c. A general professional partnership is not required to file a return of its income because it is
tax-exempt
d. Partners engaging in business as partners in a general professional partnership shall be liable
for income tax only in their separate and individual capacities

9. Statement I: All individual taxpayers, except NRA-NETB, are subject 20 % final tax on interests from
any currency bank deposit and yield or any other monetary benefit from deposit substitutes and
from trust funds and similar arrangements.

Statement II: All individual taxpayers, except NRA-NETB, are subject to 10% final tax on royalties,
in general and 20% final tax on books, literary works and musical composition.

Statement III: All individual taxpayers are subject to 20% final tax on prizes and winnings, in
general.
a. True; True; False
b. False; True; True
c. True; False; False
d. False; False; False

10. One of the following individuals is not subject to income tax in the Philippines
a. An American visiting professor who occasionally teaches at one of the leading universities in
the Philippines, and stays here for an aggregate of 185 days
b. A Filipino residing and working in America who comes to visit his relatives
c. A Korean singer who performs in one of the hotels in the Philippines for two (2) weeks, and
then leaves after he is through with his performances
d. A Japanese engineer who makes the Philippines his residence because of the nature of his
work
Use the following information for the next three (3) questions:

Pinnacle Corporation, a domestic corporation, was incorporated in 2015 but secured actual
registration from the BIR to operate in 2016.

In 2022, it reported the following:


• Gross sales of P2,000,000
• Cost of sales amounts to P1,200,000
• Operating expenses of P815,000, inclusive of P15,000 contribution to government priority
plans
• Recovered from bad debts account amounts to P40,000
• Refund of income tax amounts to P25,000

Compute for the following:

11. Total gross income of the corporation for the year


a. P800,000
b. P825,000
c. P840,000
d. P865,000

12. Total deductible expenses for the year


a. P800,000
b. P802,000
c. P804,000
d. P815,000

13. Amount of income tax payable by the corporation for the year is
a. P7,500
b. P8,400
c. P15,000
d. P16,800

14. The records of a resident citizen revealed the following income (all amounts at gross) from the
Philippine sources for the taxable year 2022:

Interest on bank deposit P100,000


Royalty from patented invention 150,000
Royalty from musical composition 75,000
Prize won in a singing contest 10,000
Share from a business partnership 50,000
Share from a general professional partnership 200,000
Dividend from domestic corporation 25,000
Lotto winnings 80,000

The total income taxes withheld amount to


a. P65,000
b. P67,000
c. P80,000
d. P81,000

15. Which of the following is not an inherent limitation of the power of taxation?
a. Territoriality
b. Public purpose test
c. Veto power of the President
d. International comity

16. The following are similarities of the inherent power of taxation, police power and eminent domain.
Which is the exception?
a. Legislative in origin
b. Affect all persons or the general public
c. Interfere with private rights and property
d. Necessary attributes of sovereignty
17. Which is true? The Chief Officials of the BIR is composed of:
I. 1 Commissioner and 4 Assistant Commissioners
II. 1 Commissioner and 4 Deputy Commissioners
III. 1 Commissioner and 1 Deputy Commissioner
IV. 1 Commissioner, 2 Deputy Commissioners and 2 Assistant Commissioners
a. I
b. II
c. III
d. IV

18. Which of the following statements is not correct?


a. For the exercise of the power of taxation, the State can tax anything at any time
b. The provisions of taxation in the Philippines Constitution are limitations of power not grants
on taxing powers
c. Taxes may be imposed to raise revenues or to regulate certain activities within the State
d. The State can have the power of taxation only if the Constitution expressly gives it the power
to tax

19. A non-resident alien individual who shall come to the Philippines and stay therein for an aggregate
period of more than 180 days during the calendar year shall be deemed a “non-resident alien doing
business in the Philippines”
a. Only when he is actually engaged in business in the Philippines
b. When he comes to the Philippines for a definite purpose which in its nature would require an
extended stay and to that end makes his home temporarily in the Philippines, although it may
be his intention at all times to return to his domicile abroad
c. Regardless of whether or not he is actually engaged in business in the Philippines
d. When his income does not come from the performance of personal services in the Philippines

20. Statement I: Estates and trusts can deduct from their gross income the same items of deductions
authorized under the Tax Code as those allowed to individual taxpayers.

Statement II: The schedular tax rates under Section 24 (A), which are prescribed for individuals,
will be used in computing the income tax of estates and trusts.

Statement III: The amount of income of the estate for the taxable year, which is properly paid or
credited during such year to any legatee, heir or beneficiary, is a special item of deduction from
the gross income of the estate.

Statement IV: An allowance paid to a widow or heir out of the corpus or principal of the estate is
deductible from the gross income of the estate.
a. Only one (1) of the above statements is true
b. Only two (2) of the above statements are true
c. Three (3) of the above statements are true
d. All of the above statements are true

21. Statement I: If the property sold is subject to a mortgage which is assumed by the buyer, and the
mortgage exceeds the basis of the property sold, the excess shall form part of the initial payments
in which case, the contract price is equal to selling price less mortgage assumed.

Statement II: Initial payment means the payment which the seller receives before or upon
execution of the instrument of sale plus other payments which he expects or is scheduled to
receive in cash or property during the taxable year of sale or disposition.

Statement III: Tips or gratuities paid directly to an employee by a customer of the employer which
are not accounted for by the employee to the employer are considered as taxable income.

Statement IV: Holiday pay and hazard pay are exempt from income tax.
a. Only one (1) of the above statements is true
b. Only two (2) of the above statements are true
c. Three (3) of the above statements are true
d. All of the above statements are true
Use the following information for the next three (3) questions:

Mr. X, a vice president of a consulting company, earned an annual gross compensation income of
P1,700,000 in 2022. In addition, he earned 13th month pay and other benefits amounting to
P50,000. The total mandatory contributions to SSS, Pag-ibig and Philhealth amounted to P10,000.

Aside from employment income, he owns a café with the following financial results:

Gross sales P2,600,000


Sales discounts 120,000
Sales returns and allowances 80,000
Cost of sales 1,200,000
Operating expenses 400,000
Non-operating income 80,000

22. If the 8% income tax rate option is availed of by Mr. X, being a mixed income earner, the income
tax payable for 2022 is
a. P583,400
b. P595,400
c. P602,400
d. P611,400

23. If Mr. X opted to be taxed at the progressive income tax rates under Sec. 24 (A) of the Tax Code,
the income tax payable for 2022 is
a. P659,600
b. P672,400
c. P723,600
d. P736,400

24. If the taxable year is 2023 and Mr. X opted to be taxed at the progressive income tax rates under
Sec. 24 (A) of the Tax Code, the income tax payable for 2023 is
a. P595,400
b. P573,500
c. P672,400
d. P723,600
25. The following items are excluded from gross income, except
a. Separation pay received by a retiring employee beyond his control
b. Retirement benefits received under RA 7641
c. Compensation for injuries or sickness
d. 13th month pay and other benefits regardless of the amount

26. Which of the following fringe benefits if given to managerial employee is exempt from fringe
benefit tax
a. Household personnel, such as driver, maid and others
b. Holiday and vacation expenses
c. Contribution of the employer for the benefit of the employee to retirement, insurance and
hospitalization benefit plan
d. Educational assistance to the employee or his dependents

Use the following information for the next two (2) questions:

ABC Corporation reported the following figures on net income, bad debts, write-offs and recoveries:

2022 Case 1 Case 2 Case 3


Net income (loss) before write off P80,000 P40,000 (P20,000)
Less: Bad debts written off 50,000 60,000 70,000
and claimed as deduction
Net income (loss) after write-off P30,000 (P20,000) (P90,000)

2023
Subsequent recovery P50,000 P50,000 P50,000

27. The taxable recovery is


Case 1 Case 2 Case 3
a. P50,000 P50,000 P50,000
b. P50,000 P0 P0
c. P50,000 P40,000 P20,000
d. P50,000 P30,000 P0

28. If the subsequent recovery for Case 1 – P30,000, Case 2 – P30,000 and Case 3 – P30,000, the taxable
recovery is
Case 1 Case 2 Case 3
a. P30,000 P30,000 P30,000
b. P30,000 P30,000 P0
c. P30,000 P20,000 P0
d. P30,000 P10,000 P0
Use the following information for the next two (2) questions:

For taxable year 2022, Mr. X signified his intention to be taxed at 8% income tax rate on gross sales in
his first quarter return. His gross sales and other information per quarter are shown below.

1st quarter 2nd quarter 3rd quarter 4th quarter


Gross sales P800,000 P1,200,000 P1,500,000 P1,300,000
Cost of sales 400,000 500,000 800,000 600,000
Operating expenses 100,000 250,000 300,000 200,000

29. How much is the income tax payable for the first quarter?
a. P12,000
b. P32,000
c. P44,000
d. P64,000

30. How much is the income tax payable for the second quarter?
a. P32,000
b. P40,000
c. P76,000
d. P96,000

31. How much is the income tax payable for the third quarter?
a. P95,000
b. P120,000
c. P140,000
d. P260,000

32. How much is the income tax payable for the final return?
a. P150,000
b. P364,000
c. P384,000
d. P385,000

33. If the taxable year is 2023, how much is the income tax payable for the final return?
a. P80,000
b. P125,000
c. P150,000
d. P384,000
34. Which of the following is an income subject to final tax?
a. Fringe benefits given to rank and file employees
b. Share of an individual taxpayer from the distributable net income of an ordinary partnership
c. Dividend income from a domestic corporation by a resident foreign corporation
d. All of the above are subject to final tax

35. For income tax purposes, the source of income is important for the taxpayer who is a
a. Filipino citizen residing in Manila
b. An American who is married to a Filipina and residing in their family home located in Taguig
City, who obtained Filipino citizenship by naturalization
c. Non-resident Filipino citizen working and residing in Australia
d. Domestic corporation

36. Statement I: A joint venture for undertaking construction projects is not taxable as a corporation.

Statement II: A consortium for energy operations pursuant to an operating consortium agreement
under a service contract with the government is not taxable as a corporation.

Statement III: A general partnership in trade is not taxable as a corporation.

Statement IV: A co-ownership where the activities of the co-owners are limited to the preservation
of property and collection of income from the property is taxable as a corporation.
a. Only one (1) of the above statements is true
b. Two (2) of the above statements are true
c. Three (3) of the above statements are true
d. All of the above statements are true

37. Which of the following statements is not correct?


a. The election of Optional Standard Deduction is irrevocable for the taxable year for which the
choice is made
b. The optional standard deduction is not available against compensation income arising out of
an employer-employee relationship
c. The optional standard deduction is an amount equal to forty percent (40%) of the Gross
income from business or practice of profession of the taxpayer
d. Unless the taxpayer signifies in his return his intention, he shall be considered as having
availed of the itemized deduction

38. The monetary value of this fringe benefit is 50% of the value of the fringe benefit
a. The employer purchases the car on installment in the name of the employee
b. The employer purchases the motor vehicle in the name of the employee
c. The employer provides the employee with cash for the purchase of a motor vehicle in the
name of the employee
d. The employer owns and maintains a fleet of motor vehicles for the use of the business and
the employees

39. In 2023, ABC Corporation provided a 3-day vacation in Baguio to all of its employees of which 80%
are rank and file. Total expenses incurred by the company for the said vacation amounted to
P1,000,000. The fringe benefit tax expense is
a. P53,846
b. P94,118
c. P107,692
d. P538,462

40. Which of the following prizes and awards shall not be subject to income tax?
a. Amount received as an award for special services, such as award for suggestions for
improving water filtration system
b. Prize in a literary contest where the recipient did not join the contest nor required to render
substantial future services as condition to receiving the prize or award
c. Contest awards of prizes received from an employer
d. Prize won in a competitive contest conducted for non-commercial or commercial purposes
41. The following rules as to recognition of capital gains or losses from the disposition of personal
property classified as capital asset apply where the taxpayer is an individual. Which is the
exception?
a. Capital losses are deductible only to the extent of the capital gains; hence, the net capital loss
is not deductible
b. Depending on the holding period, the percentages of gain or loss is 100% if the capital asset
has been held for 12 months or less; and 50% if the capital asset has been held for more than
12 months
c. Ordinary losses are deductible from capital gains; net capital losses are deductible from
ordinary gains
d. Net capital loss carried over from the previous year should not exceed the net capital gain in
the current year

Use the following information for the next four (4) questions:

Mr. X, a resident citizen, presented to you the following items of income for the taxable year 2023:

Salary as professor, net of creditable withholding tax of P1,416 P18,584


Honorarium as member of the board of directors of ABC Corporation 10,000
Rental income from apartment house 12,000
Interest income from deposit maintained under the expanded
foreign currency deposit system 4,500
Interest income on notes receivable 15,000
Interest credited on savings account with BPI 3,000
Professional fees received as a CPA 25,000
Royalty income from book publications 30,000
Prizes received from contest won 12,000
Winnings from town lottery 9,000
Mid-year bonus 18,000
13th month pay and year-end bonus 45,000
Compensation for injuries suffered in an accident 20,000

42. Compute the amount of income subject to final tax


a. P46,500
b. P49,500
c. P54,000
d. P58,500

43. Compute the amount of income excluded from gross income


a. P35,000
b. P83,000
c. P108,500
d. P141,500

44. Compute for the gross compensation income


a. P30,000
b. P88,000
c. P108,000
d. P113,000

45. Compute the business income to be included in the gross income


a. P12,000
b. P40,000
c. P42,000
d. P52,000
46. Which of the following statements is correct?
I. A minimum corporate income tax (MCIT) of 1 % of gross income from July 1, 2020 to June 30,
2023 and 2% beginning July 1, 2023 is imposed upon any domestic corporation and
nonresident foreign corporation beginning on the 4th taxable year immediately following the
taxable year in which such corporation commenced its business operations.
II. MCIT shall be imposed whenever such corporation has zero or negative taxable income, or
when the amount of MCIT is greater than normal income tax due from such corporation.
III. The computation and the payment of MCIT, shall likewise be apply at the time of filing the
quarterly corporate income tax.
a. I and II only
b. I and III only
c. II and III only
d. I, II, and III

Use the following information for the next five (5) questions:

A resident citizen has the following data on income and expenses for 2022:
Gross business income P500,000
Business expenses 200,000
Interest from savings deposit, BDO Makati, Philippines 50,000
Prize in literary contest he joined 100,000
Prize received for achievement in literature (did not join the contest) 10,000
Gain from sale of bonds (maturity is 6 years) 5,000
Separation pay from his former job (resigned) 250,000
Cash inherited from his uncle 300,000
Proceeds of his wife’s life insurance (revocable beneficiary) 1,000,000
Amount received as return of premium (premium paid, P150,000) 200,000
Tax informer’s reward 500,000
Interest income from government bonds 20,000
Winnings from illegal gambling 10,000

47. How much is the total amount of excluded or exempted income?


a. P465,000
b. P1,450,000
c. P1,460,000
d. P1,465,000

48. How much is the total final tax from certain income?
a. P30,000
b. P80,000
c. P84,000
d. P134,000
49. How much is the taxable net income subject to Section 24 (A)?
a. P500,000
b. P550,000
c. P610,000
d. None of the above

50. How much is the total income tax expense for the year?
a. P166,500
b. P82,500
c. P48,800
d. None of the choices

51. If the taxable year is 2023, how much is the total income tax expense for the year?
a. P148,500
b. P166,500
c. P82,500
d. None of the choices

52. Statement I: Since taxation presupposes an equivalent form of compensation, there should be a
direct and proximate advantage received by any taxpayer before he could be required to pay tax.

Statement II: Compensation under police power is the intangible feeling of contribution to the
general welfare of the people.

Statement III: People pay taxes which their government uses to expand its powers and territorial
domination.

Statement IV: People demand from their government certain responsibilities and then provide the
government with the means to carry them out.
a. Only one (1) of the above statements is true
b. Two (2) of the above statements are true
c. Three (3) of the above statements are true
d. All of the above statements are true

53. Which of the following transactions is exempt from capital gains tax?

Case 1: Sale of the principal residence of the taxpayer where the entire proceeds is used to
purchase a vacation lot in Tagaytay.
Case 2: Sale of a beach lot of the taxpayer where the entire proceeds is used to construct his
principal residence.
Case 3: The sale of the principal residence of the taxpayer availing tax exemption for the second
time in ten (10) years to purchase another principal residence.

a. b. c. d.
Case 1 Yes Yes No No
Case 2 Yes Yes No No
Case 3 Yes No Yes No

54. Which of the following statements is incorrect?


a. If at any time during a given taxable year, a taxpayer’s gross sales or receipts exceeded the
VAT threshold (P3,000,000), he/she shall automatically be subjected to the graduated rates
under Section 24(A)(2)(a) of the Tax Code, as amended
b. A non-VAT registered taxpayer who initially opted to avail of the 8% option but has exceeded
the VAT threshold during the taxable year, shall be subject to 3% option Percentage Tax on
the first P3,000,000 of his/her gross sales/receipts under Section 116 of the Tax Code, as
amended, with imposition of penalty even if payment is timely made on the following month
when the threshold is breached
c. The excess of the threshold shall be subject to VAT prospectively, and the 8% income tax
previously paid shall be credited to the Income Tax Due under the graduated, rates provided
in Section 24(A)(2)(a) of the Tax Code, as amended
d. None of the choices
55. The net operating loss of the business or enterprise for any taxable year immediately preceding
the current taxable year, which had not been previously offset as deduction from gross income
shall:
a. Be carried over as a deduction from gross income for the next 3 consecutive taxable years
immediately following the year of such loss
b. The business or enterprise which incurred net operating loss for taxable years 2020 and 2021
shall be allowed to carry over the same as deduction from its gross income for the next five
(5) consecutive taxable years immediately following the year of such loss
c. The net operating loss for 2020 and 2021 taxable years may be carried over as a deduction
even after the expiration of RA No. 11494 (also known as Bayanihan Act II) provided, the
same are claimed within the next five (5) consecutive taxable years immediately following
the year of such loss
d. All of the above

56. Pinnacle University (PAU) granted a scholarship grant abroad (Master of Science in Accountancy)
to Mr. X, one of its regular faculty members. The management of PAU believed that the university
will be able to improve its accountancy program through the scholarship grant. Nonetheless, no
written contract was executed between PAU and Mr. X requiring the latter to remain in the employ
of the employer for a certain period. The scholarship grant is
a. De minimis benefit, hence, tax-exempt regardless of rank
b. The benefit was provided for the benefit of the employer, hence, non-taxable on the part of
the employee
c. Subject to basic income tax or fringe benefit tax depending on the ranking of Mr. X
d. None of the above

Use the following information for the next five (5) questions:

Mr. Accounting Co. has the following information for taxable year 2023:

Quarter RCIT MCIT CWT


First 200,000 160,000 40,000
Second 240,000 500,000 60,000
Third 500,000 150,000 80,000
Fourth 300,000 200,000 70,000

Additional information:
● Excess MCIT from 2022: P60,000
● Excess tax credits from 2022: P20,000

57. How much was the income tax payable for the first quarter?
a. P80,000
b. P120,000
c. P160,000
d. P200,000

58. How much was the income tax payable for the second quarter?
a. P160,000
b. P200,000
c. P460,000
d. P660,000

59. How much was the income tax payable for the third quarter?
a. P120,000
b. P140,000
c. P600,000
d. P860,000

60. How much was the annual income tax payable?


a. P230,000
b. P390,000
c. P930,000
d. P1,260,000
61. Using the same data in the preceding problem except that the MCIT on the 4 th Quarter was
P500,000, how much was the annual income tax payable?
a. P330,000
b. P360,000
c. P380,000
d. P1,310,000

62. Which of the following statements regarding dividends is incorrect?


a. Taxable subject to regular income tax if received by a resident citizen from a non-resident
corporation
b. Taxable subject to final tax if received by a non-resident citizen from a non-resident foreign
corporation
c. Exempt from regular income tax if received by a resident foreign corporation from a domestic
corporation
d. Exempt from regular income tax if received by a domestic corporation from another domestic
corporation

63. Which of the following is NOT one of the changes under the CREATE Law?
a. IAET of 10% on improperly accumulated earnings has been repealed
b. The tax on PCSO winnings for NRA-ETB shall be 20% final withholding tax
c. Effective July 1, 2020 to December 31, 2023, the other percentage tax shall be 1% then back
to 3% afterwards
d. Foreign-sourced dividends received by a domestic corporation shall be exempt if
requirements under Sec. 27D are met

64. Which of the following is NOT one of the changes under the CREATE Law?
a. Regional operating headquarters shall be treated as a regular resident foreign corporation
starting January 1, 2022
b. Offshore banking units shall be taxed at 25% income tax rate
c. Capital gains tax on sale of unlisted shares for all types of corporations shall be at 15%
d. Domestic and foreign corporations may qualify as MSMEs if their net taxable income does
not exceed P5M and total assets, excluding land, do not exceed P100M

65. Which of the following is NOT one of the requisites for tax exemption on foreign-sourced dividends
received by a domestic corporation?
a. Funds from such dividends actually received or remitted into the Philippines are reinvested
in the business operations of the domestic corporation in the Philippines
b. The domestic corporation has held the shareholdings for a minimum of 2 years at the time
of dividend distribution
c. The domestic corporation holds directly at most 20% of the outstanding shares of the foreign
corporation
d. All of the above are requisites for tax exemption
Use the following information for the next five (5) questions:

For taxable year 2023, the net income per books after tax of CPA Corporation is P850,000, after
considering among others:

Interest income from bank deposit P5,500


Inter-corporate dividends 5,000
Gain from sale of unlisted shares of a domestic corporation 7,000
Loss from wash sate 2,500
Bad debts written off 6,500
Write-off of inventories lost due to spoilage or expiry 12,000
Depreciation on appraised value of property 15,000
Surcharge and compromise paid in relation to the late filing of ITR 80,000
Contribution to government exclusively for public purpose 50,000
Contribution to government's priority program in education 10,500
Quarterly income tax payments 65,000
Provision for bad debts 8,000

The net income per books should be reconciled with the provisions of the Tax Code, meaning, items
which are not taxable must be excluded, and items which are not deductible are to be added back.

Requirement: Assuming the total assets of the entity amounted to P110,000,000, excluding land, solve
the following:

66. How much is the net income before charitable and other contributions?
a. P1,004,000
b. P1,005,050
c. P1,008,500
d. P1,069,000

67. How much is the taxable income?


a. P943,500
b. P940,050
c. P1,005,050
d. P1,008,500

68. How much is the corporate income tax still due?


a. P186,263
b. P187,125
c. P251,263
d. P252,125

69. How much is the total final tax on income not subject to regular income tax?
a. P0
b. P1,100
c. P2,150
d. P3,150

70. How much is the total tax expense?


a. P187,363
b. P188,413
c. P189,275
d. P254,275

- - - END - - -
PINNACLE REVIEW SCHOOL, OPC
1st Pre-Board Examinations
May 2023
x----------------------------------------x

I. Obligations (14)

1. Legal compensation shall not be proper in three of the


following cases. Which is the exception?
A. Commodatum.
B. Civil liability arising from a criminal offense.
C. Gratuitous support.
D. Mutuum.

2. Consider the following: I. In alternative obligation, the


choice belongs only to the debtor, while in a facultative
obligation, the choice may be given to the creditor; II. In
alternative obligation, there is only one obligation, but
there is a substitute, while in facultative obligation, there
are many obligations; and, III. In alternative obligation,
obligation is extinguished by the fulfillment of only one of
the obligations, while in facultative obligation, the
obligation is extinguished by the fulfillment of the
principal obligation when there is no substitution yet.
A. All are true
B. All are false
C. Only Two are true
D. Only two are false

3. Rain and Brad met in SM Manila for negotiations regarding the


sale of a Maserati Levante (Luxury SUV) with plate number
POGI-004. Brad, who is a registered voter of Taguig City,
agreed to buy the Luxury SUV from Rain, who hails from Pasay
City, by tendering Php5,000,000. During a subsequent meeting
at Brad’s principal place of business in Quezon City, it was
agreed delivery will be therein on Brad’s birthday. On the
agreed date of delivery, Brad, who was at the office, called
Rain on his freshly unboxed iPhone 12 to demand delivery of
the said Luxury SUV. Rain answered and set out to comply.
When Rain answered the phone, he, in violation of the Anti-
distracted Driving Act, was driving the Luxury SUV somewhere
in in Makati City on his way to Brad’s domicile. In compliance
with the demand, where should Rain bring the Maserati Levante?
A. Pasay City
B. Makati City
C. Manila City
D. Quezon City

Page 1 of 39
PINNACLE REVIEW SCHOOL, OPC
1st Pre-Board Examinations
May 2023
x----------------------------------------x

4. D is bound to deliver a custom made Blackpink cake to C for


her birthday. The obligation was perfected on January 01,
20X0. It became due on June 30, 20X0 but D did not deliver.
Because of last night’s non-stop partying (see: inebriated),
C realized the non-compliance of D when she woke up in the
afternoon on July, 01, 20X0. However, she only communicated
the extrajudicial demand on July 31, 20X0. D delivered the
cake on August 01, 20X0. In computing legal interest, if any,
at what date should the time component be counted from?
A. July 01, 20X0
B. July 31, 20X0
C. June 30, 20X0
D. No legal interest is due

5. X, Y, Z, solidary debtors, are liable to solidary creditors


A, B, and C, in the amount of Php2,700. B condoned the entire
obligation in favor of X who accepted the renunciation. The
condonation is without the consent of A and C. Which is
CORRECT?
A. The entire obligation is extinguished.
B. Only Php900 of the obligation is extinguished.
C. Only Php300 of the obligation is extinguished.
D. The condonation of the obligation is ineffective without
the permission of A and C.

Page 2 of 39
PINNACLE REVIEW SCHOOL, OPC
1st Pre-Board Examinations
May 2023
x----------------------------------------x

6. D owes C the following debts: Php6,000.00 due on January 1;


Php6,000.00 due on January 15; Php6,000.00 due on January 31;
Php6,000.00 due on February 15; Php6,000.00 due on February
28; and Php6,000.00 due on March 15. All the debts are
unsecured except the one due on February 28. The parties
stipulated that C may demand payment on or before the due
date of each debt. Assume that today is February 16. D,
however, has not yet paid any of the debts. He has in
possession of Php3,000.00 which he intends to pay to C.
A. D may apply the payment to the debt due on February 28
because it is the most burdensome to him.
B. D may apply the payment to any of the five unsecured
debts.
C. D may not apply the payment to any of the four debts
that have become due as of today, February 16.
D. D may apply the payment to all of the debts that have
become due as of today, February 16, proportionately at
Php750.00 each.

7. Rain showed Brad a piece of Louis Vuitton Luggage and offered


to sell at a price of Php250,000. Brad agreed and the two
executed the sale in writing, performing their respective
obligations immediately. After getting home however, Brad
discovered that Rain delivered a class-A replica. In this
situation, what is the remedy available to Brad?
A. File an action for damages because Rain is guilty of
incidental fraud or dolo incidente.
B. File an action for annulment of contract because Rain is
guilty of casual fraud or dolo causante which vitiated
Brad’s consent.
C. File an action for declaration of nullity of the contract
because the subject matter of the sale is counterfeit,
therefore illegal.
D. Fila an action for rescission of contract because it is
intended to defraud the creditor, namely, Brad.

Page 3 of 39
PINNACLE REVIEW SCHOOL, OPC
1st Pre-Board Examinations
May 2023
x----------------------------------------x

8. A, B and C wrote and signed a promissory note which states


“We promise to pay X Php3,000.” At the maturity date of the
note, X wrote a letter to B condoning his obligation. How
much can X collect from A?
A. Php3,000
B. Php1,500
C. Php1,000
D. Php2,000

9. A, B and C wrote and signed a promissory note which states “I


promise to pay X, Y and Z Php9,000.” At the maturity date of
the note, how much may X collect from A?
A. Php9,000
B. Php4,500
C. Php1,000
D. Php3,000

10. The following are obligations with a term of period,


except:
A. D to give C P50,000.00 this coming December 31, 202X.
B. D to give C P50,000.00 on C’s next birthday.
C. D to give C P50,000.00 upon improvement D’s of financial
capacity.
D. D to give C P50,000.00 if C’s father dies within 2 years.

11. Which of the following condonations will not extinguish


the debtor’s obligation?
A. The condonation is in a public instrument. The acceptance
is made orally. The condonation involves payment of for a
pair of Nike Cortez shoes worth Php4,000.00.
B. The condonation and the acceptance are in a public
instrument. The condonation involves the parcel of land
within Rizal Park where the Rizal Monument is situated.
C. The condonation and the acceptance are made orally with
the creditor delivering to the debtor simultaneously the
promissory note amounting to Php10,000.00.
D. The condonation is in public instrument. The acceptance is
made in a private instrument. The condonation involves a
Bose QuietComfort Earbuds worth Php20,000.00

Page 4 of 39
PINNACLE REVIEW SCHOOL, OPC
1st Pre-Board Examinations
May 2023
x----------------------------------------x

12. Lester, 25 years old, Brad, 35 years old, and Rain, 17


years old, are solidary debtors of Jason in the amount of
P9,000.00. On due date, Jason demands payment. How much can
he collect from Lester?
A. Jason may collect from Lester 9,000.00
B. Jason may collect from Lester 6,000.00
C. Jason may collect from Lester 1,000.00
D. Jason may collect nothing because the obligation is
voidable, Rain being a minor.

13. D borrowed from C Php500,000.00. The obligation is


secured by a chattel mortgage on D’s 2010 Toyota Fortuner.
Subsequently, D paid C Php200,000.00. Unknown to D, X, a third
person, pays C Php500,000.00 believing that D still owed C
such amount.
A. X can recover Php500,000.00 from D. If D cannot pay, X can
foreclose the mortgage on D’s 2010 Toyota Fortuner.
B. X can recover Php200,000.00 from C, even if he (X) paid
without the knowledge and consent of D.
C. X can recover Php300,000.00 from D. If D cannot pay, X can
foreclose the mortgage on D’s 2010 Toyota Fortuner.
D. X can recover Php500,000.00 from C, because payment was
made without knowledge and consent of D.

14. Which of the following is not considered a conditional


obligation?
A. D to pay C Php5,000.00 if C does not walk to the moon.
B. D to pay C Php5,000.00 if D marries Y within the year.
C. D to pay C Php5,000.00 if C tops the CPA Examinations.
D. D to pay C Php5,000.00 if X does not get infected by Covid-
19 this year.

Page 5 of 39
PINNACLE REVIEW SCHOOL, OPC
1st Pre-Board Examinations
May 2023
x----------------------------------------x

II. Contracts, Sales, and Credit (28)

15. B forced X to sell him his ring. X sued for annulment,


but B had already lost the ring due to a fortuitous event. Is
B liable for loss?
A. B is not liable because the loss was without his fault.
B. B is liable to pay damages if he cannot replace the ring.
C. B is not liable to because no one shall be responsible for
the loss of a thing due to fortuitous event.
D. B is liable for the loss even if it was due to fortuitous
event because of his bad faith.

16. Consider the following: Statement I. Where the illegal


contract is a criminal offense, the innocent party shall be
bound to comply with his promise if he has not yet given
anything and if he had given, he may not claim or recover for
its return; and, Statement II. Where the illegal contract is
not a criminal offense, the innocent party may demand the
return or recover what he has given with the obligation to
comply with his promise.
A. True and True
B. False and False
C. True and False
D. False and True

17. D borrowed Php30,000 from C. In order to secure the


fulfillment of the contract of loan. D pledged and delivered
his specific laptop worth Php20,000 to C as collateral for
his obligation. They further agreed that if the loan is unpaid
at maturity date the laptop will no longer be returned to D
and will serve as payment. D defaulted in the payment of the
obligation. Thereafter, C did not return the laptop as agreed
upon without any objection from D. Which is CORRECT?
A. The said agreement is pactum commissorium and therefore
null and void.
B. Ownership of the laptop was acquired by C and the
contract of loan is fully extinguished by dacion en pago.
C. Ownership of the laptop was acquired by C by reason of
pactum commissorium.
D. The contract of loan is extinguished only up to the
extent of the value of the laptop.

Page 6 of 39
PINNACLE REVIEW SCHOOL, OPC
1st Pre-Board Examinations
May 2023
x----------------------------------------x

18. Lester posted on Facebook an invitation to bid for his


specific house and lot at a floor price of Php10,000,000.
Brad sent a private message to Lester to tender his bid at a
grossly inadequate price of Php200,000. Lester promptly
rejected the bid. Thereafter, Brad personally went to Lester
and told him a heart-breaking story. After hearing the series
of unfortunate events endured by Brad, Lester undertook to
transfer to Brad all existing rights upon the house and lot
free of charge, provided he (Brad) will shoulder the relevant
property (in arrears) and transfer (prospective) taxes as
well as other related costs on the agreement. The agreement
is sealed with a handshake, and was confirmed via a lengthy
and public Facebook Post which had at least 500k likes and
shares. What is the status of the contract?
A. Voidable
B. Rescissible
C. Void
D. Unenforceable

19. DOM, a very dirty old man, found the profile of W on


social media. After a slew of proposals, W, a young woman in
her 20s, finally met an offer she could not refuse. She agreed
to live in one of DOM’s luxury condos in the city and tend to
his needs whenever he was there. In exchange, DOM promised to
give W a monthly allowance of Php500,000. This arrangement
was to continue until she reaches the age of 40, thereupon
the agreement becomes functus officio. At this time, both M
& W are single and without any impediment to marry. The
arrangement was good while it lasted. However after 3 years,
M found a younger woman, X, offered her better terms, and
suddenly stopped giving the stipulated allowance to W (see:
ghosting). Which statement is true given the circumstances?
A. W can demand the continuation of the monthly allowance from
DOM pursuant to their agreement.
B. DOM can demand the return of the allowance that he had
given to W for the past 3 years.
C. There is no remedy, and both M and W will be criminally
prosecuted.
D. The agreement between DOM and W is against morals but does
not constitute a crime.

Page 7 of 39
PINNACLE REVIEW SCHOOL, OPC
1st Pre-Board Examinations
May 2023
x----------------------------------------x

20. X, who knew that his ring was embellished with glass,
told Y that it embellishment was emerald. Y, who knew that
his watch was gold-plated, told X that it was made of pure
gold. Y, believing that X’s ring was embellished with emerald,
and X, believing that Y’s watch was made of pure gold, then
entered into a contract whereby they exchanged their
respective articles. A week later, Y discovered that the ring
was adorned only with an ordinary glass.
A. The contract may be annulled at the instance of Y since
he discovered the fraud.
B. The contract may be annulled at the instance of X since Y
also employed fraud.
C. The contract is void because of the bad faith of both
parties; hence, it shall not produce any effect.
D. Neither party may ask for annulment since both are guilty
of fraud. The contract, therefore, is valid.

21. TL went to the store of C and asked the latter to sell


to him construction materials on credit. Since C did not know
him, he denied TL’s request. LF, a rich businessman in the
community, happened to be at the store at that time and
overheard the conversation. He told C, “I personally know TL,
don’t worry, a Lannister always pays his debt.” Accordingly,
C granted credit sale to TL. The sale was evidenced by an
invoice issued by C. Later, however, C could not collect from
TL because it turned out that representation made by LF as to
the credit of TL was false.
A. C can go after LF for damages by reason of the latter’s
misrepresentation.
B. C cannot go after LF for damages because LF’s
representation was not in writing.
C. C can go after LF because there was a sales invoice
evidencing the sale.
D. C can go after LF because it was LF’s representation that
induced him to grant a credit sale to TL, the form of the
representation is immaterial.

Page 8 of 39
PINNACLE REVIEW SCHOOL, OPC
1st Pre-Board Examinations
May 2023
x----------------------------------------x

22. Six years ago, Rain, who was 17 years old, sold his
agricultural lot to Lester for Php500,000. Lester bought the
said land for planting maize or corn as he was a transitioning
“plantito.” On the other hand, Rain, driven by his
entrepreneurial spirit, invested the proceeds of said sale in
a Kpop-themed restaurant business which was initially
extremely lucrative but had gone through difficult times as
a result of the current economic crisis brought by the Covid-
19 pandemic. He thought of the agricultural lot that he had
sold to Lester, whom he heard had recently finally managed to
make use of the land via bumper crops (particularly palay,
because the soil was completely unfit to support either maize
or corn). Rain, realizing his grave mistake, now approaches
you to seek counsel on how to get the lot from Lester. What
advice will you give to Rain?
A. Rain may annul the sale but not Lester.
B. Lester may annul the sale but not Rain.
C. Neither Rain nor Lester may annul the sale.
D. Either Rain and Lester may annul the sale.

23. Consider the following: Statement I – If at the time of


making the offer or acceptance one of the parties was insane,
the resulting contract is voidable; Statement II - If before
the acceptance is conveyed to the offeror, either of the
parties becomes insane, the resulting contract is void; and,
Statement III - If after the acceptance is conveyed to the
offeror, either of the parties becomes insane, the resulting
contract is valid.
A. All statements are incorrect.
B. All statements are correct.
C. Only two statements are correct.
D. Only two statements are incorrect.

Page 9 of 39
PINNACLE REVIEW SCHOOL, OPC
1st Pre-Board Examinations
May 2023
x----------------------------------------x

24. Reformation of an instrument is available in the


following cases, except:
A. When a mutual mistake of the parties causes the failure
of the instrument to disclose their agreement.
B. When one party was mistaken, and the other party acted
fraudulently or inequitably in such a way that the
instrument does not show their true intention.
C. When a party was mistaken and the other knew or believed
that the instrument did not state their real agreement.
D. When one of the parties has brought an action to enforce
the contract.

25. A sold in writing to X his old macbook for Php 30,000.


However, after the execution of the deed of sale, there was
never any delivery from A and nor payment of the price from
X. Given the circumstances, the contract is:
A. Void
B. Unenforceable
C. Rescissible
D. Voidable

26. S orally sold to B his house at 139 Maginhawa St., Quezon


City. Out of excitement, in the written deed of sale, both
became careless about the true number of the house and instead
wrote, “193 Maginhawa St., Quezon City,” which was another
house also owned by S. In this case, the remedy shall be:
A. Annulment of a voidable contract because of mutual mistake.
B. Reformation of instrument because of mutual error.
C. Specific performance because all the elements of the
contract are present.
D. Declaration of nullity of the contract because of the
uncertainty of the intention as to the object.

Page 10 of 39
PINNACLE REVIEW SCHOOL, OPC
1st Pre-Board Examinations
May 2023
x----------------------------------------x

27. To defraud his creditor, A sold his house to X. When


however the creditor wanted to collect his credit, somebody
lent A enough money to pay. Is the sale rescissible?
A. Yes, because it was entered into in fraud of creditors.
B. No, because the creditor can collect the credit due him.
C. No, because the debtor has become in good faith when he
was lent enough money to pay his debts.
D. Yes, because the debtor was in bad faith when he sold the
house to X.

28. To defraud his creditors, A sold his real property to B.


B now seeks to register the sale. X, a creditor, seeks to
prevent the registration on the ground that it is a
rescissible contract. The creditor (X) promptly filed and
caused to be annotated and adverse claim upon the title of
the real property. May the land be registered in B’s name?
A. No, because the contract is rescissible and therefore
without force and effect.
B. No, because the sale is voidable and after annulment is no
longer binding.
C. Yes, because the contract although voidable is still valid
and binding before annulment.
D. Yes, because the contract although rescissible is valid,
binding and enforceable before rescission.

29. B orally leased to X his go-cart for 368 days. No


delivery of the go-cart was made, and no payment of the
rentals was given. B now refuses to pay the rentals in advance
as agreed upon. Decide.
A. B may rightfully refuse because the contract is
unenforceable.
B. B cannot refuse because the contract is enforceable.
C. X cannot compel B to do something against the latter’s
will.
D. The contract is reciprocal and therefore B cannot be
compelled to pay because there was no delivery of the go-
cart.

Page 11 of 39
PINNACLE REVIEW SCHOOL, OPC
1st Pre-Board Examinations
May 2023
x----------------------------------------x

30. Can future inheritance be the subject of contract of


sale?
A. No, since it will put the predecessor at the risk of harm
from a tempted buyer, contrary to public policy
B. Yes, since the death of the decedent is certain to occur
C. No, since the seller owns no inheritance while his
predecessor lives
D. Yes, but on the condition that the amount of inheritance
can only be ascertained after the obligations of the estate
have been paid

31. D mortgaged his land to C as security for a loan. Fearing


foreclosure of the mortgage due to his inability to pay the
loan, D sold the land to B without the consent of C. Which is
correct?
A. The sale is void unless D can give another security
B. B cannot acquire ownership over the land even if D delivers
the land to him
C. D cannot sell the land if there is an agreement prohibiting
the mortgagor from alienating the land
D. D can sell the land even without the consent of C

32. S and B signed a deed of sale of specific motor vehicle


at a price of P200,000. The parties agreed that S would
deliver the said car and B would pay the price after two
weeks. Before the two week period was over, S found another
buyer who was willing to pay P250,000 for the car. May S
validly withdraw from his contract with B?
A. Yes because B has not yet paid the price making the contract
unenforceable
B. Yes because the contract of sale is not yet perfected
without payment of earnest money
C. No, because S was already bound but his contract with B
and he cannot unilaterally withdraw from it
D. Yes because S has not yet delivered the car

Page 12 of 39
PINNACLE REVIEW SCHOOL, OPC
1st Pre-Board Examinations
May 2023
x----------------------------------------x

33. X pledged her ring to Y to secure a P10,000 obligation


payable in two years. One year thereafter, Y gives note in
writing to X stating that the debt need not be secured and
that X may get the ring back at her most convenient time. As
a result,
A. The principal obligation and the contract of pledge are
both extinguished
B. Only the principal obligation is extinguished but not the
contract of pledge because X did not get back the ring
C. Both the principal obligation and the contract of pledge
are not extinguished because X did not get back the ring
D. The pledge is extinguished and Y is constituted as a
depository

34. A, B and C are co-owners of a rural lot with an area of


1,000 sqm in the ratio of 6:1:3, respectively. The co-owned
lot is bound by the farm lots of D and E with areas of 50 sqm
and 75 sqm, respectively. A sold his share of the co-owned
lot to X. Who shall have the preferred right to repurchase
the lot sold by A to X?
A. B because his share in the co-owned lot is smaller
B. B and C with an area of 150 sqm and 450 sqm, respectively
C. D because his adjoining rural lot has smaller area
D. E because his adjoining rural lot has larger area

35. Consider the following: Statement I - X owns 50 mango


trees bearing fruits, ready for harvest. She sold all the
fruits of all the trees to Y who paid P100,000.00. X told to
Y that she can harvest all the fruits anytime she likes and
pointing at the mango trees. For legal purposes, X has
fulfilled her obligation to deliver the mango fruits to Y by
traditio longa manu; and, Statement II - B rented the specific
truck of S. After the end of the contract, S sold the property
to B. As a result of sales, B continues to have possession of
the property. In this contract of sale, the constructive
delivery is present is traditio brevi manu.
A. Both are true
B. Both are false
C. Statement 1 is true while Statement 2 is false
D. Statement 1 is false while Statement 2 is true

Page 13 of 39
PINNACLE REVIEW SCHOOL, OPC
1st Pre-Board Examinations
May 2023
x----------------------------------------x

36. A bought pair of shoes from a shoe store and a repair


shop. It was later discovered, however, that the shoes did
not belong to the store but to a customer who had left the
same for repair. Did A acquire good title to the shoes?
A. No, because it was sold by the store which is not the owner
thereof
B. No, because it is the fault of A in not exercising diligence
in buying the shoes as to its real ownership
C. Yes, because it was bought from a store in good faith for
value
D. Yes, because the owner of the shoes was precluded from
setting up the want of authority of the store in selling
the shoes

37. S sold to B a dog on January 10, 2021. Two days after


delivery, the dog suffered some illness which caused its
death. B is now suing S for breach of warranty against hidden
defects because the dog died within three days after delivery.
Decide
A. S is liable because the disease existed within three days
after purchase
B. S is not liable if he was not aware of the disease, which
caused the death of the dog
C. S is liable if he were in bad faith
D. S is not liable because the disease did not exist at the
time of purchase

38. On 05 July 20X1 A sold for P2,000,000 her house and lot
to B. it was agreed that delivery of the house and lot, and
the payment therefore would be made on 01 August 20X1.
Unfortunately, C negligently set the house on fire on 26 July
20X1 and the house was completely destroyed. Which is correct?
A. B is not required to pay the P900,000 since the contact
had no subject matter
B. A must still deliver the lot but is excused from delivering
the house, while B must still pay the P2,000,000
C. A must still deliver the lot while B should pay only the
amount equivalent to the value of the lot
D. A need not deliver the lot while B need not to pay the
P2,000,000

Page 14 of 39
PINNACLE REVIEW SCHOOL, OPC
1st Pre-Board Examinations
May 2023
x----------------------------------------x

39. X borrowed money from Y and gave a piece of land as


security by way of mortgage. It was agreed between the parties
that upon non-payment of the loan, the land would already
belong to Y. If X failed to pay the debt, would Y now become
the owner of the land?
A. Y would become the owner because it was agreed upon by them
based on the principle of autonomy of contracts
B. Y would not become the owner because the agreement that he
would become the owner upon default of X is against the
law
C. Y would become the owner but with right of redemption by X
D. Y would not become owner if X annuls the avoidable
agreement

40. Consider the following: Statement I – If the same car


should have been sold to different buyers, the ownership shall
be transferred to the person who may first taken possession
thereof in good faith; and, Statement II – The seller is
responsible to the buyer for any hidden defects or faults in
the thing sold only if he was aware thereof
A. Statement I is true, Statement II is false
B. Both statements are false
C. Statement I is false, Statement II is true
D. Both statements are true

41. A mortgaged his car to B for P200,000. A failed to pay


his obligation. B sold it at public auction for P180,000. Can
B recover the deficiency?
A. Yes, even without stipulation
B. Yes, only if there is stipulation
C. No, even if there is stipulation
D. No, unless there is stipulation

Page 15 of 39
PINNACLE REVIEW SCHOOL, OPC
1st Pre-Board Examinations
May 2023
x----------------------------------------x

42. Consider the following: Statement I - In a true pacto de


retro sale, the title and ownership of the property sold are
immediately vested in the vendee a retro subject only to the
resolutory condition of repurchase by the vendor a retro
within the stipulated period known as conventional
redemption; and, Statement II - The right of legal redemption
refers to the right of an adjoining owner of an urban land to
be given preference to the purchase of a small piece of urban
land which is held for speculation before it is offered for
sale to others
A. Both are true
B. Both are false
C. Statement 1 is true while Statement 2 is false
D. Statement 1 is false while Statement 2 is true

Page 16 of 39
PINNACLE REVIEW SCHOOL, OPC
1st Pre-Board Examinations
May 2023
x----------------------------------------x

III. Intellectual Property Code (5)

43. Madulas Ltd., Inc. wishes to aggressively advertise its


unique and special frying pans (specially coated using the
world-renowned “Dudulas” formula) both on traditional and
alternative media using 20-second sketches and songs. It
wants to make sure that nobody will use the video nor audio,
as well as the words of the songs. What type of IP protection
should it get?
A. Patent
B. Trade Secret
C. Copyright
D. Trademark

44. Madulas Ltd., Inc. wishes to enhance its sales for its
frying pans not only by using the world-renowned “Dudulas”
formula, but also by giving them a more appealing, aesthetic
and aerodynamic shape. The company wants to make sure that it
has to sole use of the new appearance or look. What type of
IP protection should it get?
A. Patent
B. Utility model
C. Industrial design
D. Trademark

45. Two marks for the same product are currently being
examined by the Trademark Examiner of the Intellectual
Property Office to determine if they are confusingly similar.
One product bears the trademark “Big Mak” while the other
product uses the trademark “Big Mac.” If the case is filed
prior to 2021, what test shall be applied to determine if
there is trademark infringement?
A. Dominancy test
B. Holistic test
C. Enabling test
D. Best mode test

Page 17 of 39
PINNACLE REVIEW SCHOOL, OPC
1st Pre-Board Examinations
May 2023
x----------------------------------------x

46. Two marks for the same product (Single Malt Scotch) are
currently being examined by the Trademark Examiner of the
Intellectual Property Office to determine if they are
confusingly similar. The names of the manufactures are
apparently different as printed on the bottom part of their
labels. However, the label designs, colors, print styles and
sizes are very similar and almost identical. If the case is
filed prior to 2021, what test shall be applied to determine
if there is trademark infringement?
A. Dominancy test
B. Holistic test
C. Enabling test
D. Best mode test

47. This defines the scope of protection given to the owner


of the patent; it is treated separately for purposes of
determining patent validity and infringement.

A. Patent Protection
B. Patent Coverage
C. Patent Claims
D. Patent Scope

Page 18 of 39
PINNACLE REVIEW SCHOOL, OPC
1st Pre-Board Examinations
May 2023
x----------------------------------------x

IV. SRC and CorpGov (7)

48. Any person who acts as intermediary in making deliveries


upon payment to effect settlement in securities transactions
is a/an:
A. Pre-need plans
B. Exchange
C. Clearing agency
D. Insider

49. The requirement of registration shall not as a general


rule apply to any of the following classes of securities:
A. Any security issued or guaranteed by the Government of the
Philippines, or by any political subdivision or agency
thereof, or by any person controlled or supervised by, and
acting as an instrumentality of said Government.
B. Any security issued or guaranteed by the government of any
country with which the Philippines maintains diplomatic
relations, or by any state, province or political
subdivision thereof on the basis of reciprocity: Provided,
that the Commission may require compliance with the form
and content for disclosures the Commission may prescribe
C. Certificates issued by a receiver or by a trustee in
bankruptcy duly approved by the proper adjudicatory body
D. All of the above

50. It shall be unlawful for an insider to sell or buy a


security of the issuer, while in possession of material
information with respect to the issuer or the security that
is not generally available to the public, unless: Statement
I: The insider proves that the information was not gained
from such relationship; and, Statement II: If the other party
selling to or buying from the insider (or his agent) is
identified, the insider proves: (a) that he disclosed the
information to the other party, or (b) that he had reason to
believe that the other party otherwise is also in possession
of the information
A. Only Statement I is true
B. Only Statement II is true
C. Both are true
D. Both are false

Page 19 of 39
PINNACLE REVIEW SCHOOL, OPC
1st Pre-Board Examinations
May 2023
x----------------------------------------x

51. The company should disclose all relevant information on


its corporate governance policies and practices in the Annual
Corporate Governance Report (ACGR), which should be submitted
to the Commission, and continuously updated and posted on the
company’s website. The ACGR should contain the following
disclosures, among others, except:
A. A policy on disclosure of all relevant and material
information on individual board directors and key
executives to evaluate their experience and qualifications
B. Board and executive remuneration, as well as the level and
mix of the same
C. Accurate disclosure to the public of every material fact
or event that occurs in the company
D. All of the above are required ACGR disclosures

52. The Board should designate a lead director among the


independent directors if the Chairman of the Board is not
independent, including if the positions of the Chairman of
the Board and CEO are held by one person. The functions of
the lead director, among others are the following, except:
A. Serves as an intermediary between the Chairman and the
other directors when necessary
B. Convenes and chairs meetings of the non-executive directors
C. Contributes to the performance evaluation of the Chairman,
as required
D. All of the above are functions of the lead director

Page 20 of 39
PINNACLE REVIEW SCHOOL, OPC
1st Pre-Board Examinations
May 2023
x----------------------------------------x

53. Consider the following: Statement I: Securities


deposited pursuant to a tender offer or request or invitation
for tenders may be withdrawn by or on behalf of the depositor
at any time throughout the period that tender offer remains
open and if the securities deposited have not been previously
accepted for payment, and at any time after 60 days from the
date of the original tender offer to request or invitation,
except as the Commission may otherwise prescribe; Statement
II: Where any person varies the terms of a tender offer or
request or invitation for tenders before the expiration
thereof by increasing the consideration offered to holders of
such securities, such person shall pay the increased
consideration to each security holder whose securities are
taken up and paid for whether or not such securities have
been taken up by such person before the variation of the
tender offer or request or invitation.
A. Only Statement I is true
B. Only Statement II is true
C. Both are true
D. Both are false

54. To avoid conflict or a split board and to foster an


appropriate balance of power, increased accountability and
better capacity for independent decision-making, it is
recommended that the positions of Chairman and Chief
Executive Officer (CEO) be held by different individuals.
Which of the following is not a role and responsibility of
the CEO?
A. Determines the corporation’s strategic direction and
formulates and implements its strategic plan on the
direction of the business
B. Communicates and implements the corporation’s vision,
mission, values and overall strategy and promotes any
organization or stakeholder change in relation to the same
C. Oversees the operations of the corporation and manages
human and financial resources in accordance with the
strategic plans
D. All of the above are roles and responsibilities of the
CEO

Page 21 of 39
PINNACLE REVIEW SCHOOL, OPC
1st Pre-Board Examinations
May 2023
x----------------------------------------x

V. Batas Pambansa Blg. 22 (5)

55. Consider the following: Statement I: What B.P. 22


punishes is the issuance of a bouncing check and the purpose
for which the check was issued; Statement II: What B.P. 22
punishes is the issuance of a bouncing check and not the non-
payment of an obligation; and, Statement III: The Rule that
there is only one offense when the offender is moved by one
criminal intent cannot apply to B.P. 22.
A. Only two are true
B. Only two are false
C. All are true
D. All are false

56. What is the quantum of evidence required for conviction


in a criminal case for violation of B.P. 22?
A. Preponderance of evidence
B. Proof beyond reasonable doubt
C. Prima facie evidence or probable cause
D. Clear and convincing evidence

57. Consider the following: Statement I: It shall not be the


duty of the drawee of any check, when refusing to pay the
same to the holder thereof upon presentment, to cause to be
written, printed, or stamped in plain language thereon, or
attached thereto, the reason for drawee’s dishonor or refusal
to pay the same; and, Statement II: When there are no
sufficient funds in or credit with such drawee bank, such
fact shall always be explicitly stated in the notice of
dishonor or refusal
A. Only I is true
B. Only II is true
C. Both are true
D. Both are false

Page 22 of 39
PINNACLE REVIEW SCHOOL, OPC
1st Pre-Board Examinations
May 2023
x----------------------------------------x

58. Consider the following: Statement I: A corporate check


requires two (2) signatories. If said check was allegedly
issued bearing only one (1) signature and was subsequently
dishonored, only the persons who actually signed will be
liable for B.P. 22; and, Statement II: A check was issued to
a kidnapper as payment for ransom. If the check was thereafter
dishonored due to insufficient funds, the drawer can be made
liable for violation of B.P. 22.
A. Only I is true
B. Only II is true
C. Both are true
D. Both are false

59. What is the quantum of evidence required for the filing


of a criminal case for violation of Batas Pambansa Bilang 22
a.k.a Anti-Bouncing Checks Law?
A. Preponderance of evidence
B. Proof beyond reasonable doubt
C. Prima facie evidence or probable cause
D. Clear and convincing evidence

Page 23 of 39
PINNACLE REVIEW SCHOOL, OPC
1st Pre-Board Examinations
May 2023
x----------------------------------------x

VI. Secrecy of Bank Deposits (5)

60. A government employee purchased Philippine Treasury


retail bonds with a total value of Php100,000.00 and receives
annual interest income from his investment. Under Republic
Act No. 1405, the law on secrecy of bank deposits, the
investment of the government employee may be inquired into:
A. Public prosecutor handling estafa case against the
government employee
B. BIR officer in charge of auditing the income tax
liabilities of the government employee
C. Credit investigator of a lending institution where the
government employee applied for the loan
D. None of the foregoing

61. Which is FALSE about Foreign Currency Deposit Act also


known as RA No. 6426 as amended?
A. The foreign currency deposits maintained abroad by
Filipino citizens are not covered by the provisions of
this act.
B. Depositary banks which have been qualifies by the Monetary
Board to function under expanded foreign currency deposit
system are exempt in maintain the fifteen percent (15%)
foreign currency cover for their liabilities in the form
of foreign currency deposit with BSP.
C. Foreign currency deposits are of absolutely confidential
nature, except upon the written permission of the
depositor.
D. Foreign currency deposits are subject to examination in
cases of impeachment, or upon order of a competent court
in cases of bribery or dereliction of duty of public
officials.

Page 24 of 39
PINNACLE REVIEW SCHOOL, OPC
1st Pre-Board Examinations
May 2023
x----------------------------------------x

62. Lester was waiting in for his turn to deposit money


inside ABC Bank. When it was his turn, he saw a Rain, a
prominent social media influencer, had just finished his
transaction and intentionally left a crumpled a piece of paper
on the bank teller’s countertop. After a few moments of being
starstruck, he proceeded with depositing Php5,000,000.00.
Once he received his deposit slip, he took the crumpled piece
of paper and placed it in his pocket, unnoticed by the teller,
so he could later throw it away. As he was walking towards
the door, he needed to answer a phone call.

When he got home, he noticed that he had forgotten to throw


away the crumpled paper in his pocket. He opened it up and
saw the amount of Php100,000,000.00 deposited by Rain and his
corresponding bank account number. Lester immediately took a
photo and sent it to a Group Chat composed of at least 10
people.

As Rain was scrolling on social media, he saw his bank account


details on a post by a group called “Yown!” with the hashtag
#PaScholarNamanDiyan being shared by a person called Brad.

Who is in violation of R.A. 1405 (Secrecy of Bank Deposits)?

A. The teller of the Bank, because he did not object to the


taking of the crumpled paper.
B. Lester, because his act of taking a photo and subsequent
sharing was the proximate case of the disclosure.
C. The people on the Group Chat, the group “Yown!,” and Brad
as they had actively revealed the bank account details.
D. None of the foregoing.

Page 25 of 39
PINNACLE REVIEW SCHOOL, OPC
1st Pre-Board Examinations
May 2023
x----------------------------------------x

63. X, a private individual, maintains US Dollar deposit


account with ABC Bank. X is suspected to be the leader of a
notorious assassination syndicate and is believed to have
deposited all money received as compensation for such
services in said deposit account. The authorities want to
open the said account to know to verify if there are really
deposits in big amounts. Which statement is most accurate?
A. An approval from the Monetary Board is necessary to open
the account
B. The same rules under the law on Secrecy of Bank Deposits
will apply
C. Because the deposit is in US Dollars, it is covered by
the Foreign Currency Deposit Act which allows disclosure
only upon the written permission of the depository
D. Approval from the Court is necessary to order disclosure
of the account

64. Which of the following is an exception to the law on


secrecy of bank deposits in Philippine Pesos, but NOT an
exception to the secrecy of foreign currency deposits?
A. Upon Banko Sentral ng Pilipinas (BSP) inquiry into or
examination of deposits or investments with any bank, when
the inquiry or examination is made in the course of the
BSP’s periodic special examination of said bank to ensure
compliance with the Anti-Money Laundering Act (AMLA)
B. Upon inquiry of Philippine Deposit Insurance Corporation
(PDIC) inquiry into and examination of deposit accounts in
case there is a finding of unsafe or unsound banking
practice
C. Upon subpoena of the Senate of the Philippines, sitting as
an impeachment court, at the trial of the accused after
impeachment by the House of Representatives
D. None of the foregoing.

Page 26 of 39
PINNACLE REVIEW SCHOOL, OPC
1st Pre-Board Examinations
May 2023
x----------------------------------------x

VII. PDIC (5)

65. The following are the main functions of PDIC except:


A. Deposit insurer
B. Receiver/liquidator of distressed banks
C. Financial Intelligence Agency
D. Co-regulator of the BSP over banking institutions

66. X maintains the following deposits at Pinnacle Bank:


Php 600,000 – savings deposit in the name of X ITF A
Php 100,000 – time deposit in the name of ABC Partnership and
X
Php 400,000 – money market account in the name of X
How much can X personally recover from PDIC if Pinnacle Bank
was liquidated by BSP?
A. Php 550,000
B. Php 1,050,000
C. Php 500,000
D. Zero

67. Which of the following statements is FALSE regarding


payment of insured deposit by PDIC
A. Payment may be made in cash or through making available to
each depositor a transferred deposit in another insured
bank in an amount equal to insured deposit of such
depositor.
B. The claim for insured deposit should be settled within six
(6) months from the date of filling provided all
requirements are met but the claim must be filed within
twenty four (24) months after bank takeover.
C. The six-month period shall apply even if the documents of
the claimant are incomplete or if the validity of the claim
requires the resolution of issues of facts and law by
another office, body or agency, independently or in
coordination with PDIC.
D. None of the above.

Page 27 of 39
PINNACLE REVIEW SCHOOL, OPC
1st Pre-Board Examinations
May 2023
x----------------------------------------x

68. X has the following deposit accounts:

X Bakery Checking BPI-Ortigas Php300,000


Business (Sole Account Branch
Proprietorship)
X Savings BDO- Php400,000
Deposit Muntinlupa
Branch
X and Y Time Deposit BPI-Makati Php600,000
Branch
X and Z Savings BDO-Quezon Php800,000
Deposit City Branch

How much is the total amount which may be covered by deposit


insurance in so far as X is concerned?

A. Php1,000,000
B. Php550,000
C. Php2,000,000
D. Php1,200,000

69. Statement I: A bank’s Inter-branch deposits are covered


by deposit insurance

Statement II: Membership of banks to PDIC is mandatory; hence,


all operating banks in the Philippines, including local and
overseas branches, are members of PDIC

A. True, True
B. True, False
C. False, True
D. False, False

Page 28 of 39
PINNACLE REVIEW SCHOOL, OPC
1st Pre-Board Examinations
May 2023
x----------------------------------------x

VIII. Anti-Money Laundering Act (14)

70. Who is the chairman of the Anti-Money Laundering Council


A. BSP Governor
B. Insurance Commissioner
C. SEC Chairperson
D. BIR Commissioner

71. What is the quantitative threshold for a covered


transaction in by real estate developers and brokers?
A. A single transaction exceeding Php7,500,000 in cash
B. A single transaction exceeding Php7,500,000 in cash or cash
equivalents
C. A single transaction exceeding Php7,500,000 in cash or on
account
D. A single transaction exceeding Php7,500,000 on account

72. Which of the following is considered covered transaction


under AMLA for a banking institution?
A. Over Php250,000 or more in one banking day
B. Php500,000 or more in one banking day
C. Over Php500,000 in one banking day
D. Php1,000,000 or more in one banking day.

73. A petition for civil forfeiture of any monetary


instrument or property is in any way related to an unlawful
activity or a money laundering offense must be filed with
the:
A. Court of Appeals
B. Sandiganbayan
C. Regional Trial Court
D. Supreme Court

74. What is the quantitative threshold for a covered


transaction in a casino?
A. Amount equal to Php5,000,000 per day
B. Amount below Php5,000,000 per day
C. Amount exceeding Php5,000,000 per day.
D. Any amount per day.

Page 29 of 39
PINNACLE REVIEW SCHOOL, OPC
1st Pre-Board Examinations
May 2023
x----------------------------------------x

75. Which of the following statements about prosecution of


money laundering offenses is FALSE?
A. The prosecution of money laundering and the unlawful
activity shall proceed independently such that any person
may be charged with and convicted of both money laundering
and the unlawful activity
B. The elements of money laundering are separate and distinct
from the element of the unlawful activity
C. The elements of the unlawful activity, including the
identity of the perpetrators and the details of the
commission of the unlawful activity, need not be
established by proof beyond reasonable doubt in the case
for money laundering
D. None of the above

76. Which of the following is a predicate crime of Money


Laundering Offense?
A. Theft
B. Sedition
C. Libel
D. Tax Evasion

77. No administrative, criminal, or civil proceedings, shall


lie against any person for having made a covered transaction
report of a suspicious transaction report in the regular
performance of his duties and in good faith, whether or not
such reporting results in any criminal prosecution under this
Act or any other Philippine law.
A. Safe harbor provision
B. Prosecutorial Immunity
C. Nulla Poena Sine Lege
D. These all mean the same thing

78. Which of the following is a predicate crime of Money


Laundering Offense?
A. Video voyeurism
B. Sexual Assault
C. Unjust Vexation
D. Sexual harassment

Page 30 of 39
PINNACLE REVIEW SCHOOL, OPC
1st Pre-Board Examinations
May 2023
x----------------------------------------x

79. What is the period allowed by AMLA for the Court of


Appeals to resolve the verified ex parte petition filed by
the AMLC for the issuance of freeze order on bank accounts
related to proceeds of any unlawful activity?
A. Within 12 hours from the filing of the petition
B. Within 24 hours from the filing of the petition
C. Within 36 hours from the filing of the petition
D. Within 48 hours from the filing of the petition

80. The following are predicate crimes that may authorize


Anti-Money Laundering Council to inquire bank accounts even
without court order from Court of Appeals, except:
A. Destructive arson
B. Murder
C. Hijacking
D. Hacking

81. Consider the following: Statement I: Covered


institutions shall maintain accounts only in the true name of
the account owner or holder. The provisions of existing laws
to the contrary notwithstanding, anonymous account, accounts
under fictitious names, incorrect name and all other similar
accounts shall be absolutely prohibited; and, Statement II:
When a covered institution acquires the business of another
covered institution, either in whole or as a product
portfolio, it is not necessary for the identity of all
existing customers to be re-established: Provided, that all
customer account records are acquired with the business and
due diligence inquiries do not raise any doubt as to whether
or not the acquired business has fully complied with all the
requirements under the AMLA and these Rules.
A. Statement I is true, statement II is false
B. Both statements are true
C. Statement I is false, statement II is true
D. Both statements are false

Page 31 of 39
PINNACLE REVIEW SCHOOL, OPC
1st Pre-Board Examinations
May 2023
x----------------------------------------x

82. Within what period must the AMLC file in the Court of
Appeals a verified ex parte petition for the issuance of
freeze order for purposes of implementing targeted financial
sanctions in relation to proliferation of weapons of mass
destruction and its financing?
A. Within 6 months from actual or constructive knowledge
B. Within 24 hours from actual or constructive knowledge
C. Immediately from actual or constructive knowledge
D. None of the above

83. Which of the following is necessary or condition


precedent before the commencement or resolution of petition
for civil forfeiture of any monetary instrument or property
is in any way related to an unlawful activity or a money
laundering offense?
A. Prior criminal charge of unlawful activity
B. Prior conviction of unlawful activity
C. Prior criminal charge of money laundering offense
D. None of the above

Page 32 of 39
PINNACLE REVIEW SCHOOL, OPC
1st Pre-Board Examinations
May 2023
x----------------------------------------x

IX. Data Privacy Act (7)

84. It refers to any act of information relating to a natural


or juridical persons to the extent that, although the
information is not processed by equipment operating
automatically in response to instructions given for that
purpose, the set is structure, either by reference to
individuals or by reference to criteria relating to
individuals, in such a way that specific information relating
to a particular person is readily accessible.
A. Internal Control system
B. Information system
C. Equipment system
D. Filing system

85. It refers to any operation or any set of operations


performed upon personal information including, but not
limited to, the collection, recording, organization, storage,
updating or modification, retrieval, consultation, use,
consolidation, blocking, erasure, or destruction of data.
A. Operations
B. Organization
C. Destruction
D. Processing

86. Data Privacy Act applies:


Statement I: To the processing of only certain types of
personal information.

Statement II: Only to natural persons involved in personal


information processing including those personal information
controllers and processors who, although not found or
established in the Philippines, use equipment that are
located in the Philippines, or those who maintain an office,
branch or agency in the Philippines.
A. True, True
B. True, False
C. False, True
D. False, False

Page 33 of 39
PINNACLE REVIEW SCHOOL, OPC
1st Pre-Board Examinations
May 2023
x----------------------------------------x

87. Data Privacy Act does not apply to:


Statement I: Information about an individual who is or was
performing service under contract for a government
institution that relates to the services performed, including
the terms of the contract, and the name of the individual
given in the course of the performance of those services.

Statement II: Information relating to any discretionary


benefit of a financial nature such as the granting of a
license or permit given by the government to an individual,
including the name of the individual and the exact nature of
the benefit.
A. True, True
B. True, False
C. False, True
D. False, False

88. The processing of personal information shall be


permitted only if not otherwise prohibited by law, and when
at least one of the following conditions exists. Which is not
one of the conditions?
A. The processing of personal information is necessary and is
related to the fulfillment of a contract with the data
subject on in order to take steps at the request of the
data subject prior to entering into a contract.
B. The processing is unnecessary for compliance with a legal
obligation to which the personal information controller is
subject.
C. The processing is necessary to protect vitally important
interests of the data subject, including life and health.
D. None of the above.

Page 34 of 39
PINNACLE REVIEW SCHOOL, OPC
1st Pre-Board Examinations
May 2023
x----------------------------------------x

89. Consider the following: I. DOH has no right to release


information on COVID-19 cases that are relevant for public
health interventions which may include details on a patient’s
sex, age, barangay, travel history, etc, even in such a way
that a COVID-19 suspected, probable, or confirmed case should
not be capable of being identified from the data that is
released; and, II. A University receives, processes, and
resolves complaints involving its students, faculty members
and administrative personnel, and in the course of such
proceedings and up until their conclusion, it can allow
access to some or all information relating to such proceedings
to various parties such as the complainant, respondent,
witnesses and other parties who may be affected by the case
or its outcome. Which is/are true?
A. I Only
B. I & II
C. II Only
D. None are true

90. Statement I: A personal information controller may


subcontract the processing of personal information.

Statement II: Personal information controllers may not invoke


the principle of privileged communication over privileged
information that they lawfully control or process.
A. True, True
B. True, False
C. False, True
D. False, False

Page 35 of 39
PINNACLE REVIEW SCHOOL, OPC
1st Pre-Board Examinations
May 2023
x----------------------------------------x

X. Electronic Commerce Act (5)

91. Brad, using his own laptop, sent an email to Lester


offering to give, propter nuptias, his only Condominium in
Proscenium at Rockwell. Lester read the said email on his
cell phone. Thereafter, Lester acknowledged the previous
email and accepted the offer of Brad via direct message on
Brad’s Instagram account. Given the circumstances:

Statement I: The contract is not valid as to both Brad and


Lester.

Statement II: The elements of the contract are not in the


form of an electronic document.
A. True, True
B. True, False
C. False, True
D. False, False

92. Consider the following: Statement I: An electronic data


message or electronic document is deemed to be dispatched at
the place where the originator has its place of business and
received at the place where the addressee has its place of
business any agreement to the contrary notwithstanding; and,
Statement II: Unless otherwise agreed between the originator
and the addressee, the dispatch of an electronic data message
or electronic document occurs when it enters an information
system outside the control of the originator or of the person
who sent the electronic data message or electronic document
on behalf of the originator.
A. True, True
B. True, False
C. False, True
D. False, False

93. It refers to a person who is intended by the originator


to receive the electronic data message or electronic document
A. Sender
B. Addressee
C. Recipient
D. None of the above

Page 36 of 39
PINNACLE REVIEW SCHOOL, OPC
1st Pre-Board Examinations
May 2023
x----------------------------------------x

94. Consider the following: Statement I: Hacking by


definition under RA No. 8792 includes the introduction of
viruses to a computer or information and communication
system; and, Statement II: Piracy by definition under RA No.
8792 is identical to infringement as defined in RA No. 8293
A. True, True
B. True, False
C. False, True
D. False, False

95. In any legal proceedings, nothing in the application of


the rules on evidence shall deny the admissibility of an
electronic data message or electronic document in evidence:
Statement I: On the sole ground that it is in electronic
form

Statement II: On the ground that it is not in the standard


written form
A. True, True
B. True, False
C. False, True
D. False, False

Page 37 of 39
PINNACLE REVIEW SCHOOL, OPC
1st Pre-Board Examinations
May 2023
x----------------------------------------x

XI. Ease of Doing Business (5)

96. The process or transaction between applicants or


requesting parties and government offices or agencies
involving applications for any privilege, right, reward,
license, clearance, permit or authorization, concession, or
for any modification, renewal or extension of the enumerated
applications or requests which are acted upon in the ordinary
course of business of the agency or office concerned.
A. Front Line Service
B. Civil Service
C. Citizen’s Charter
D. Government Service

97. These applications or requests submitted by applicants


or requesting parties of a government office which
necessitate evaluation in the resolution of complicated
issues by an officer or employee of said government office,
requiring the use of technical knowledge, specialized skills
and/or training.
A. Simple transactions
B. Complex transactions
C. Highly technical transactions
D. None of the above

98. The following are the requisites of immunity under the


Ease of Doing Business and Efficient Government Service
Delivery Act of 2018, except
A. There is absolute necessity for the testimony of the
respondent/accused-informant whose discharge is requested
B. There is no other direct evidence available for the
proper prosecution of the offense committed, except the
testimony of said respondent/accused-informant
C. The testimony of said respondent/accused-informant can be
substantially corroborated in its material points
D. The respondent/accused-informant appears to be the most
guilty but has not been previously convicted of a crime
involving moral turpitude

Page 38 of 39
PINNACLE REVIEW SCHOOL, OPC
1st Pre-Board Examinations
May 2023
x----------------------------------------x

99. This refers to the policy adopted by RA 11032 prohibiting


officers and employees of Government Agencies from having any
contact with an applicant or requesting party, in any manner,
concerning an application or request, except: (a) during
preliminary assessment of the request and evaluation of
sufficiency of submitted documents; and (b) when strictly
necessary.
A. Anti-Red Tape
B. Zero Contact
C. No Lunch Break
D. Anti-Fixer

100. Any individual whether or not officially involved in the


operation of a government office or agency who has access to
people working therein, and whether or not in collusion with
them, facilitates speedy completion of transactions for
pecuniary gain or any other advantage or consideration.
A. Agent
B. Broker
C. Liaison
D. Fixer

- NOTHING FOLLOWS -

Page 39 of 39
FINANCIAL ACCOUNTING AND REPORTING (FAR)
FIRST PREBOARD EXAMINATION
MAY 2023 LECPA

Direction: Choose the letter corresponding to the best answer for each of the questions provided below.
This examination consists of 70 items and the exam is good for three (3) hours.

1. In reconciling the bank balance with the book cash balance, which of the following would not cause
the bank balance shown on the bank statement to be lower than the unadjusted book balance?
a. Cash on hand at the company
b. Interest credited to the account by the bank
c. NSF checks from a customer, as reported on the bank statement
d. Deposits in transit

2. Which of the following would not be included in the inventory amount reported on a company’s
balance sheet
a. Items shipped out on consignment to another company
b. Items shipped today FOB shipping point; invoice had been mailed to the customer
c. Items in the receiving department of the company; returned by the customer, invoice has been
mailed
d. Items purchased from a supplier and en route directly to a customer of the company; the term
is FOB destination invoice received but not yet paid

3. When a periodic inventory system is used


a. Two entries must be made when goods are purchased
b. Cost of goods sold is a residual amount, rather than an account
c. Ending inventory is treated as an expense and beginning inventory is treated as an asset
d. ‘Purchases’ account is not used; all inventory purchase entries are debited to inventory account

4. Which method does not properly match expense and revenue?


a. Charging bad debts as accounts are written off as uncollectible
b. Charging bad debts with a percentage of sales under the allowance method
c. Charging bad debts using aging of accounts receivable under the allowance method
d. Charging bad debts using a percentage of accounts receivable under the allowance method

5. The failure to record a purchase of merchandise on account even though the goods are properly
included in the physical inventory results in
a. An overstatement of asset and net income
b. An understatement of asset and net income
c. An understatement of liability and an overstatement of equity
d. An understatement of cost of goods sold and liability and an overstatement of assets

6. Under the equity method of accounting for investments, an investor recognizes the share of the
earnings in the period in which the
a. Investee pays a dividend
b. Investor sells the investment
c. Investee declares a dividend
d. Earnings are reported by the investee in the financial statements
Use the following information for the next three (3) questions:

Pinnacle Company purchased a land for P2,000,000. There was a small office building on the land with
fair value of P700,000 which the entity will continue to use with some modification and renovation.
The fair value of the land is P1,300,000.

The entity decided to construct a factory building and incurred the following costs:

Materials and supplies P3,000,000


Excavation 100,000
Labor on construction 2,500,000
Cost remodeling office building 200,000
Legal cost of conveying land 10,000
Imputed interest on money used during construction 120,000
Cash discounts on materials purchased 60,000
Supervision by management 70,000
Compensation insurance premium for workers 20,000
Clerical and other expenses related to construction 30,000
Paving of streets and sidewalks 40,000
Plans and specifications 140,000
Payment for claim for injuries not covered by insurance 25,000
Legal cost of injury claim 15,000
Saving on construction 200,000

7. What is the initial cost of land?


a. P1,300,000
b. P1,310,000
c. P1,350,000
d. P1,410,000

8. What is the initial cost of office building?


a. P700,000
b. P850,000
c. P900,000
d. P1,050,000

9. What is the initial cost of factory building?


a. P5,600,000
b. P5,720,000
c. P5,800,000
d. P5,920,000

10. You obtained the following information relating to DC Company’s bank account for December 2025. In
reconciling the bank balance on December 31, 2025, you observed the following facts:

Balance per bank statement, 12/31/25 P1,565,700


Outstanding checks, 12/31/25 575,400
Proceeds of bank loan, 12/15/25, omitted from records 182,280
Deposit of 12/23/25, omitted from bank statement 147,780
Check #123 of AB, charged by the bank in error to DC 66,900
Proceeds of note receivable of DC collected by bank, 12/10/25, not entered
in cash records (Principal, P40,000; Interest, P400; Collection charge, P100) 40,300
Erroneous debit memo on 12/31/25, charged company’s account 150,000
Deposit of another client on 12/6/25 credited in error to DC 25,000

The cash balance per books of DC Company on December 31, 2025 is


a. P1,107,400
b. P1,329,980
c. P1,552,560
d. P1,565,700
11. Lester Corporation had the following items listed in its trial balance on December 31, 2025:

Currency and coins P733


Balance in checking account 50,000
Customer checks waiting to be deposited 3,300
Treasury bills, purchased on 11/2/25, mature on 4/30/26 3,000
Marketable equity securities 10,200
Commercial paper, purchased on 11/2/25, mature on 1/30/26 5,500

What amount will Lester include in its year-end statement of financial position as cash and cash
equivalents?
a. P57,033
b. P59,533
c. P69,733
d. P72,733

Use the following information for the next four (4) questions:

Situation 1: The following information pertains to Itachi Company for the year ended December 31,
2025:

Credit sales during 2025 P4,450,000


Collection of accounts written off in prior periods 170,000
Worthless accounts written off in 2025 191,000
Allowance for doubtful accounts, January 1, 2025 155,000

Itachi Company provides for doubtful accounts based on 1½% of credit sales.

Situation 2: Kisame Company received from a customer a one-year, P375,000 note bearing annual
interest of 8%. After holding the note for six months, Kisame discounted the note at Metrobank at an
effective interest rate of 10%.

Situation 3: Sasori Company included the following items in its inventory on December 31, 2025:
• Merchandise out on consignment to Father Company at sales price – P3,000,000
• Goods purchased in transit, FOB shipping point – P2,000,000
• Goods held on consignment from Mother Company – P1,000,000
• Goods sold in transit, FOB shipping point, at sales price – P4,500,000
• Freight paid by Sasori on consigned goods to Father – P100,000
• Freight paid by Mother Company – P50,000
• Freight paid by Sasori on goods purchased – P10,000
• Freight paid by Sasori on goods sold – P20,000
• Mark-up on cost is 25%. Consigned goods were still unsold on December 31, 2025

12. What amount should Itachi Company report as allowance for doubtful accounts on December 31,
2025?
a. P66,750
b. P200,750
c. P242,750
d. P345,750

13. How much did Kisame Company receive from the bank?
a. P371,429
b. P384,750
c. P392,858
d. P405,000

14. What amount of loss from discounting should Kisame Company recognize?
a. P0
b. P5,250
c. P9,750
d. P20,250
15. How much should be included in Sasori Company’s inventory on December 31, 2025?
a. P4,410,000
b. P4,510,000
c. P5,000,000
d. P5,010,000

16. Under PFRS 13, the fair value of an asset or a liability is measured as
a. The appraised value of the lease or liability
b. The cost of the asset less accumulated depreciation or the carrying amount of the liability on the
date of sale
c. The price that would be paid to acquire the asset or received to assume the liability in an orderly
transaction between market participants
d. The price that would be received when selling an asset or paid when transferring a liability in an
orderly transaction between market participants

17. Which of the following statements is true regarding capitalization of interest?


a. Interest cost capitalized in connection with the purchase of land to be used as a building site
should be debited to the land account and not the building account
b. The amount of interest cost capitalized during the period should not exceed the actual interest
cost incurred
c. When excess borrowed funds not immediately needed for construction are temporarily
invested, any interest earned should be recorded as interest revenue
d. The minimum amount of interest to be capitalized is determined by multiplying a weighted-
average interest rate by the amount of average accumulated expenditures on qualifying asset
during the period

Use the following information for the next four (4) questions:

Situation 1: On January 1, 2025, Nagato Company purchased 40% of the outstanding shares of Pain
Company, paying P6,400,000 when the carrying amount of the net assets of Pain Company equaled
P12,500,000. The difference was attributed to equipment which had a carrying amount of P2,500,000
and a fair value of P4,000,000 and building which had a carrying amount of P6,000,000 and fair value
of P9,000,000. The remaining useful life of the equipment and building was 4 years and 12 years,
respectively. During 2025, Pain Company reported net income of P5,000,000 and paid dividends of
P2,500,000.

Situation 2: Konan Company has the following property items on December 31, 2025:
• Land which at the date of acquisition is not intended for any specific use in the future –
P1,000,000
• Land held for future plant site – P2,000,000
• Building being leased out under an operating lease – P8,000,000
• Building being leased out under finance lease – P2,500,000
• Equipment being leased under finance lease – P1,500,000
• Land and building acquired under a finance lease being used by the entity as its general and
administrative headquarter – P1,500,000
• Condominium building that is being constructed intended for sale in the ordinary course of
business – P5,000,000
• Building being leased out under operating lease, an insignificant portion is used for
administrative purposes – P6,000,000
• Hotel building owned which significant services are provided to the guests – P7,000,000
Situation 3: Yahiko Company purchased land and building at a single cost of P20,000,000. It was
determined that the fair values of the land and building were P18,000,000 and P7,000,000,
respectively. The entity demolished the building to make way for construction of a new building. The
total contract price and other directly attributable cost amounted to P15,000,000. The entity also
incurred demolition cost of P350,000.
18. What amount of investment income should Nagato Company recognize for 2025?
a. P1,000,000
b. P1,750,000
c. P1,800,000
d. P2,000,000
19. How much will be reported as investment properties in Konan Company’s separate financial
statements?
a. P9,000,000
b. P15,000,000
c. P16,500,000
d. P22,000,000

20. How much will be reported as owner occupied properties in Konan Company’s separate financial
statements?
a. P10,000,000
b. P10,500,000
c. P11,500,000
d. P18,500,000

21. What total amount should be recognized as property, plant, and equipment by Yahiko Company?
a. P29,400,000
b. P29,750,000
c. P35,350,000
d. P33,350,000

22. How is a significant amount of consignment inventory reported in the statement of financial position?
a. The inventory is reported separately on the consignee’s statement of financial position
b. The inventory is reported separately on the consignor’s statement of financial position
c. The inventory is combined with other inventory on the consignee’s statement of financial
position
d. The inventory is combined with other inventory on the consignor’s statement of financial
position

23. An entity recently purchased land a usable hotel with the plan to tear down immediately the hotel and
build a new luxury hotel. The allocated cost of the old hotel should be
a. Capitalized as part of the cost of the land
b. Depreciated over the remaining useful life
c. Written off as loss in the year the hotel is torn
d. Capitalized as part of the cost of the new hotel

Use the following information for the next two (2) questions:

LECPA Company is a wholesaler of scented candles. The activity for item #123 during June is presented
below:

Date Transaction Units Cost


June 01 Inventory balance 6,000 20.00
04 Purchases 9,000 24.00
12 Sales 10,800
19 Purchases 14,400 26.00
22 Sales 11,400
29 Purchases 4,800 27.00

24. Under the FIFO periodic inventory system, how much is the ending inventory of item #123 on June 30?
a. P280,800
b. P278,400
c. P302,400
d. P316,800

25. Under the weighted average cost periodic inventory system, how much is the ending inventory of item
#123 on June 30?
a. P278,400
b. P294,720
c. P302,400
d. P316,800
26. Which of the following statements is incorrect in relation to government grant?
a. Any adjustment needed when government grant becomes repayable is accounted for as a
change in accounting estimate
b. Where conditions apply to a government grant, if should only be recognized when there is
reasonable assurance that the conditions will be met
c. In respect of loan from the government at zero interest rate, an imputed interest charge should
be made in profit or loss
d. A government grant that becomes receivable as compensation for losses already incurred should
be recognized as income of the period in which it becomes receivable

27. What is the correct valuation approach for financial asset?


a. Not held for collection at fair value and held for collection at fair value
b. Not held for collection at amortized cost and held for collection at fair value
c. Not held for collection at fair value and held for collection at amortized cost
d. Not held for collection at amortized cost and held for collection at amortized cost

Use the following information for the next four (4) questions:

Situation 1: In January 2025, Esmeralda Company purchased a mineral mine for P3,600,000 with
removal ore estimates by geographical surveys at 2,160,000 tons. The property has an estimated value
of P360,000 after the ore has been extracted. Esmeralda Company incurred P1,080,000 of
development costs preparing the property for extraction of ore. During 2025, 270,000 tons were
extracted and 240,000 were sold.

Situation 2: Khufra Company self-constructed an asset for its own use. Construction started on January
1, 2025 and the asset was completed on December 31, 2025. The company had a two-year, 18% loan
of P500,000, specifically obtained to finance the asset construction. Funds not yet needed during the
construction were temporarily invested in a short-term debt security yielding a P10,000 interest
revenue. The company also had general borrowings amounting to P600,000, 5-year term with interest
of 20% and P1,000,000, 10-year term with interest of 18% were used in part in the self-construction.
Costs incurred during the year were as follows: January 1 – P400,000; April 1 – P500,000; August 1 –
P480,000; and December 1 – P180,000.

Situation 3: Minotaur Company bought a machine for P400,000 on January 1, 2025 accounted for using
the revaluation model. The machine’s useful life is 10 years and it is estimated to have a zero residual
value and is depreciated using the straight line method. The fair value of the machine is as follows:
December 31, 2025, P475,000; December 31, 2026, P520,000; December 31, 2027, P495,000.

Situation 4: Barracks Company contracted with another entity to construct a custom-made equipment.
The equipment was completed and ready for use on January 1, 2025. Barracks Company paid the
equipment by issuing a P500,000 three-year note that specified 4% interest, payable annually on
December 31 of each year. The cash price equivalent was unknown. It was determined by comparison
with similar transactions that 12% was a reasonable rate of interest. The PV of 1 at 12% for 3 periods
is 0.71 and the PV of an ordinary annuity of 1 at 12% for three periods is 2.40.
28. What amount of depletion should Esmeralda company report for 2025?
a. P360,000
b. P480,000
c. P520,000
d. P540,000

29. What amount of interest should Khufra Company capitalize as part of the self-constructed asset?
a. P155,000
b. P165,000
c. P171,875
d. P181,875

30. What amount of revaluation surplus should Minotaur Company recognize in its December 31, 2026
statement of financial position?
a. P120,000
b. P155,000
c. P175,000
d. P200,000
31. At what amount should Barracks Company initially recognize the equipment?
a. P355,000
b. P369,200
c. P403,000
d. P500,000

Use the following information for the next four (4) questions:

Do San Company acquired all the assets and liabilities of Samsan Company. Samsan Company has a
number of operating divisions, including whose major industry is the manufacture of toy train,
particularly those having historical significance. The toy trains division is regarded as a cash-generating
unit. In paying P20,000,000 for the net assets of Samsan Company, Do San calculated that it had
acquired goodwill of P2,400,000. At the end of the reporting period, the carrying amounts of the assets
of the toy train division were:

Factory building P2,500,000


Inventory 1,500,000
Brand name 500,000
Goodwill 500,000
Total P5,000,000

There is a declining interest in toy trains because of the aggressive marketing of computer-based toys,
so the management of Do San Company measured the value in use of the toy train division at
P4,230,000.
32. Do San Company should report Factory building at what amount?
a. P2,115,000
b. P2,350,000
c. P2,400,000
d. P2,500,000

33. Do San Company should report Inventory at what amount?


a. P1,269,000
b. P1,350,000
c. P1,410,000
d. P1,500,000

34. Do San Company should report Brand name at what amount?


a. P115,000
b. P423,000
c. P470,000
d. P500,000

35. Do San Company should report Goodwill at what amount?


a. P0
b. P115,000
c. P423,000
d. P500,000
Use the following information for the next six (6) questions:

On January 2, 2025, Pinnacle Inc. acquired 60,000 ordinary shares (par value, P50) of Mr. Accounting
Corporation for a total consideration of P3,600,000 which includes P36,000 transaction costs. These
share are not intended for trading purposes. The company made an irrevocable decision to designate
these non-trading securities at fair value through other comprehensive income.

December 31, 2025, Mr. Accounting ordinary shares were selling at P58 per share.

On March 31, 2026, Mr. Accounting issued 25% ordinary share dividends. The market price of the Mr.
Accounting ordinary shares on this date was P62 per share.

On October 29, 2026, Pinnacle sold 30,000 of its Mr. Accounting ordinary shares at P57 per share.

The market price of Mr. Accounting ordinary shares on December 31, 2026 is P52 per share.

36. What amount of unrealized gain (loss) on fair value change shall be reported in the OCI section of the
statement of comprehensive income for the year 2025?
a. P84,000
b. (P84,000)
c. P120,000
d. (P120,000)

37. What amount of dividend income shall be recognized during 2026?


a. Zero
b. P180,000
c. P750,000
d. P930,000

38. What amount of gain (loss) on sale shall be reported for 2026?
a. Zero
b. P30,000
c. (P30,000)
d. P318,000

39. What amount shall be reclassified to retained earnings as a result of the sale of shares on October 29,
2026?
a. Zero
b. P270,000
c. P318,000
d. P405,000

40. At what amount should be the investment be reported on December 31, 2026?
a. P1,560,000
b. P1,740,000
c. P2,088,000
d. P2,340,000

41. What cumulative amount of unrealized gain (loss) on fair value change shall be reported on December
31, 2026?
a. P180,000
b. (P180,000)
c. P225,000
d. (P225,000)
Use the following information for the next seven (7) questions:

On January 1, 2025, CPA Company purchased four-year, P4,000,000, 10% bonds at fair value. The
prevailing market rate of interest on this date was 8%. Interest is payable annually every December 31.
The company’s business model is to collect contractual cash flows to sell the asset. The bonds were
sold on December 31, 2027 at fair value.

The prevailing market rate of interest on December 31, 2025, 2026 and 2027 are provided below:

Date Rate
December 31, 2025 9.0%
December 31, 2026 10.0%
December 31, 2027 7.0%

The PV factors are as follows:

PV of 1 at 8% for 4 periods 0.7350


PV of an ordinary annuity of 1 at 8% for 4 periods 3.3121
PV of 1 at 9% for 3 periods 0.7722
PV of an ordinary annuity of 1 at 9% for 3 periods 2.5313
PV of 1 at 10% for 2 periods 0.8264
PV of an ordinary annuity of 1 at 10% for 2 periods 1.7355
PV of an ordinary annuity of 1 at 7% for 1 period 0.9346

42. At what amount shall the investment be initially recognized on January 1, 2025?
a. P3,746,368
b. P4,000,000
c. P4,206,040
d. P4,264,840

43. At what amount shall the investment be presented on December 31, 2025?
a. P4,000,000
b. P4,070,440
c. P4,101,320
d. P4,206,040

44. How much shall be reported in the OCI section of the statement of comprehensive income for the year
2025?
a. P104,707
b. (P104,707)
c. P163,520
d. (P163,520)

45. How much is the interest income for the year 2026?
a. P328,106
b. P336,482
c. P369,119
d. P400,000

46. At what amount shall the investment be presented on December 31, 2026?
a. P3,999,800
b. P4,073,960
c. P4,142,520
d. P4,400,000

47. What cumulative amount of unrealized gain (loss) should be reported as a component of other
comprehensive income in the statement of changes in equity on December 31, 2026?
a. P38,002
b. (P38,002)
c. P142,709
d. (P142,709)
48. How much is the gain (loss) on sale to be recognized in profit or loss on December 31, 2027?
a. Zero
b. P38,330
c. (P38,330)
d. P181,039

49. Which statement is correct regarding initial recognition of research and development costs?
a. Research costs may be capitalized
b. All development costs should be capitalized
c. If an entity cannot distinguish the research phase of an internal project to create an intangible
asset from the development phase, the entity treats the expenditure for that project as if it were
incurred in the development phase only
d. Development costs may be capitalized

50. If a note receivable is discounted with recourse on a conditional sale


a. A contingent liability does not exist
b. Note receivable discounted is credited
c. Liability for note receivable discounted is credited
d. Note receivable must be credited

51. In 2025, ABC Company experienced a decline in the value of inventory resulting in a writedown from
cost of P1,800,000 to net realizable value of P1,500,000.

The entity used the allowance method to record the necessary adjustment.

In 2026, market conditions have improved dramatically. On December 31, 2026, the inventory had a
cost of P2,500,000 and net realizable value of P2,300,000.

What is included in the adjusting entry on December 31, 2026?


a. Debit gain on reversal of inventory writedown P100,000
b. Credit gain on reversal of inventory writedown P200,000
c. Debit allowance for inventory writedown P100,000
d. Credit allowance for inventory writedown P200,000

52. On January 1, 2025, ABC Company purchased a large quantity of personal computers. The cost of these
computers was P3,000,000. On the date of purchase, the management estimated that the computers
would last approximately 4 years and would have a residual value of P300,000. The entity used the
double declining balance method. During January 2026, the management realized that technological
advancements had made the computers virtually obsolete and that they would have to be replaced.
The residual value did not change. Management proposed changing the remaining life of the
computers to 2 years. What amount should ABC report a depreciation for 2026?
a. P600,000
b. P750,000
c. P1,200,000
d. P1,500,000
Use the following information for the next two (2) questions:

ABC Company reported an impairment loss of P1,500,000 in 2025. This loss was related to an equipment
acquired on January 1, 2021 for P10,000,000 with no residual value. Straight line depreciation was
recorded at P500,000 annually until 2025. Depreciation for 2026 was computed based on the recoverable
amount of the equipment on December 21, 2025. The entity decided to measure the equipment using the
revaluation model on December 31, 2027. On such date, the asset had a fair value of P9,000,000.

53. What amount of gain on reversal of impairment should be recognized in 2027?


a. P0
b. P500,000
c. P1,300,000
d. P3,800,000

54. What amount of revaluation surplus should be recognized in 2027?


a. P0
b. P1,000,000
c. P1,500,000
d. P2,500,000

55. On January 1, 2025, ABC company acquired both a License (franchise) and a Trademark in exchange
for 2,000 shares of ABC, P100 par ordinary shares. The shares are selling for P125 per share on January
1, 2025. The trademark is worth thrice as much as the license. The license may be used for five years
while the trademark has a remaining useful life of 6 years. ABC Company intends to renew the
trademark continuously because the said trademark is expected to contribute to net cash flows
indefinitely.

How much is the amortization expense for the year 2025?


a. P0
b. P12,500
c. P43,750
d. P47,916
Use the following information for the next two (2) questions:

Pinnacle Company used the average retail inventory method. On December 31, 2025, the following
information relating to the inventory was gathered:

Cost Retail
Inventory – January 1 P190,000 P450,000
Purchases 2,990,000 4,350,000
Purchase discounts 40,000
Freight in 150,000
Markups 300,000
Markdowns 400,000
Sales 4,400,000
Sales return 100,000
Sales discount 50,000
Sales allowance 30,000

56. What is the cost ratio?


a. 50%
b. 60%
c. 70%
d. 80%

57. What is the estimated cost of the inventory on December 31, 2025?
a. P400,000
b. P280,000
c. P245,000
d. P315,000

58. Jay Company provides financing to other companies by purchasing their accounts receivable on a
nonrecourse basis. Jay charges its clients a commission of 15% on all receivable factored. In addition,
Jay withholds 10% of receivables factored as protection against sales returns or other adjustments.
Experience has led Jay to establish an allowance for bad debts of 6% of all receivables purchased.

On January 15, Jay purchased receivables from Soriano Company totaling P2,500,000. Soriano had
previously an allowance for bad debts for these receivables at P35,000. By January 31, Jay had collected
P2,200,000 on these receivables. What is the loss on factoring to be recognized by Soriano Company?
a. P0
b. P340,000
c. P375,000
d. P625,000

59. During 2025, Juaquin Co. pays an insurance premium of P45,000 on a P1,000,000 life insurance policy
covering the president. The cash surrender value of the policy will increase from P170,000 to P180,200
during 2025. The entity received dividends of P10,500 from the insurance company during 2025.

The president died half-way through 2025. The policy indicates that the cash surrender value is
P175,600 at that date and 50% of the premium is refunded. The life insurance expense for the year
2025 is
a. P6,400
b. P9,000
c. P12,300
d. P24,300
60. The cash account of XYZ Corp. on December 31, 2025 has a balance of P127,600 and it consists of the
following:

Bills and coins on hand P52,780


Petty cash including petty cash vouchers of P650 1,000
Balance in savings account with a bank closed by the BSP 36,000
Customer’s check dated Jan. 15, 2026 8,000
Credit memo from suppliers for purchases returns 6,500
Postage stamps 120
Money order 800
IOU of an employee 400
Checking account balance in BDO 22,000

The correct cash balance on December 31, 2025 is


a. P75,130
b. P75,930
c. P76,580
d. P76,330

61. Which of the following costs are not included in ‘costs to sell’?
a. Transfer taxes and duties
b. Levies by regulatory agencies
c. Commissions to broker and dealers
d. Transport and other costs necessary to get the assets to a market

62. Which of the following items is an example of investment property?


a. Property that is leased to another entity under a finance lease
b. Property held for short-term sale in the ordinary course of business
c. Property that is being constructed or developed on behalf of third parties
d. Property that is being constructed or developed for future use as investment property

63. What is the measurement basis of an asset that is acquired in a non-monetary exchange?
With commercial substance With no commercial substance
a. Carrying amount of asset given up Carrying amount of asset given up
b. Carrying amount of asset given up Fair value of asset given up
c. Fair value of asset given up Carrying amount of asset given up
d. Fair value of asset given up Fair value of asset given up

64. ABC Company purchased a jewel polishing machine for P3,600,000 on January 1, 2025 and received a
government grant of P500,000 toward the capital cost. The accounting policy is to treat the grant as a
reduction in the cost of the asset.

The machine is to be depreciated on a straight-line basis over 8 years and estimated to have a residual
value of P50,000 at the end of this period. What is the depreciation of the machine for 2025?
a. P387,500
b. P381,250
c. P443,750
d. P762,500
65. The following information appears in BSA Company’s records for the year ended December 31, 2025:

Inventory, January 1 P325,000


Purchases 1,150,000
Purchase returns 40,000
Freight in 30,000
Sales 1,700,000
Sales discounts 10,000
Sales returns 15,000

On December 31, the company conducted a physical inventory which revealed that the ending
inventory was only P210,000. The gross profit on net sales has remained constant at 30% in recent
years. BSA suspects that some inventory may have been pilfered by one of the company’s employees.

How much is the estimated cost of missing inventory on December 31?


a. P75,500
b. P82,500
c. P210,000
d. P292,500

66. In relation to the amortization of intangible assets, the basic rule under PAS 38 is that unless
demonstrated otherwise
a. The residual value of does not enter into the determination of the amortization charge
b. The residual value need not be reviewed at the end of each annual reporting period
c. The residual value is presumed to be zero
d. All intangible assets have a residual value at least equal to the amount of maintenance costs
incurred

Use the following information for the next four (4) questions:

The details of Mr. Accounting Company’s “Intangible Assets” account in its financial statements for
year 2025 are shown below:

• On January 1, 2025, Mr. Accounting signed an agreement to operate as franchise of Mastery Company
for an initial franchise of P680,000. Of this amount, P200,000 was paid when the agreement was signed
and the balance was payable in four annual payments of P120,000 each, beginning January 1, 2026.
The agreement provides that the down payment is not refundable, and no future services are required
of the franchisor. The implicit rate for loan of this type is 14%. The PV factor of ordinary annuity at 14%
for 4 periods. 2.9137

The agreement also provides the 5% of the revenue from the franchise for 2025 was P8,000,000. Mr.
Accounting estimates that the useful life of the franchise to be ten years.

• Mr. Accounting incurred P624,000 of research and development costs in its laboratory to develop a
patent which was granted on January 2, 2025. Legal fees and another costs associated with the
registration of the patent totaled P131,200. Mr. Accounting estimates that the useful life of the patent
will be eight years.

• A trademark was purchased from CPA Company for P320,000 on July 1, 2022. Expenditure for
successful litigation defense of the trademark totaling P80,000 were paid on July 1, 2025. Mr.
Accounting estimates that the trademark’s useful life will be indefinite.

Determine the following:

67. Total expenses related to franchise in 2025


a. P448,950
b. P454,964
c. P503,914
d. P535,200
68. Carrying amount of franchise as of December 31, 2025
a. P494,680
b. P538,733
c. P549,644
d. P612,000

69. Carrying amount of patent as of December 31, 2025


a. P114,800
b. P123,482
c. P124,640
d. P131,200

70. Carrying amount of trademark as of December 31, 2025


a. P288,000
b. P304,000
c. P320,000
d. P400,000

- - - END - - -
ADVANCED FINANCIAL ACCOUNTING AND REPORTING (AFAR)
FIRST PREBOARD EXAMINATION
MAY 2023 LECPA

Direction: Choose the letter corresponding to the best answer for each of the questions provided below.
This examination consists of 70 items and the exam is good for three (3) hours.

1. What is the nature of liability of general partners as to partnership debts or obligations?


a. They are liable equally up to the extent of their separate assets after the partnership assets
are exhausted
b. They are liable pro-rata up to the extent of their capital contribution only
c. They are liable solidarily up to the extent of their separate assets after the partnership assets
are exhausted
d. They are liable pro-rata up to the extent of their separate assets after the partnership assets
are exhausted

2. When property other than cash is invested in a partnership, at what amount should the non-cash
property be credited to the contributing partner’s capital account?
a. Fair value at the date of contribution
b. Contributing partner’s tax basis
c. Contributing partner’s original cost
d. Assessed valuation for property tax purposes

3. In the formation of the ABC Partnership, the total of the face value of cash and fair value of non-
cash assets contributed by partner A is lower than the amount credited to his capital, which of the
following explanation is allowed by Philippine GAAP?
a. Partner A received bonus from partners B and C
b. Partner A’s contributed asset is subject to asset revaluation
c. Partner A contributed goodwill to the partnership through his special skill or industry
d. Partner A gave bonus to partners B and C

Use the following information for the next two (2) questions:

A and B are partners sharing profits and losses in the ratio of 60% and 40%, respectively. The
partnership’s balance sheet on August 30 follows:

Cash P9,720 Accounts payable P10,800


Other assets 95,760 A, loan 4,680
B, Loan 7,200 A, capital 64,800
B, capital 32,400
Total P112,680 Total P112,680

At this date, C was admitted as a partner for a consideration of P35,100 cash for a 30% interest in
capital and in profits.

4. Assume C is admitted by purchase of 30% each of the original partners’ interest, determine how
the P35,100 will be apportioned to A and B, respectively
a. A, P23,004 and B P12,096
b. A, P32,450 and B P16,300
c. A, P32,850 and B, P15,900
d. A, P32,950 and B, P15,800

5. Assume C is admitted by investing the P35,100 to the partnership, determine the effects of any
bonus over the capital balances of the original partners:
a. A, P(2,754) and B P(1,836)
b. A, P(9,900) and B, P(14,850)
c. A, P(14,850) and B, P(9,900)
d. A, P9,000 and B, P14,850
Use the following information for the next four (4) questions:

On July 1, A and B decided to form a partnership. The capitals are to be based on net assets transferred
after the following adjustments:
a. A and B’s inventory is to be valued at P62,000 and P44,000, respectively.
b. Accounts receivable of P4,000 in A’s books and P2,000 in B’s books are uncollectible.
c. Accrued salaries of P8,000 for A and P10,000 for B are still to be recognized in the books.
d. Unused office supplies of A amounted to P10,000, while that of B amounted to P3,000.
e. Unrecorded patent of P14,000 and prepaid rent of P9,000 are to be recognized in the books A
and B, respectively.
f. A is to invest or withdrew cash necessary to have a 40% interest in the firm.

Balance sheets for A and B on July before adjustments are given below:

A B
Cash P62,000 P100,000
Accounts Receivable 52,000 40,000
Inventory 64,000 48,000
Office supplies - 10,000
Equipment 40,000 48,000
Accumulated depreciation – equipment (18,000) (6,000)
Total Assets P200,000 P240,000

Accounts payable P56,000 P 40,000


Capitals 144,000 200,000
Total Liabilities and Capital P200,000 P240,000

Determine the following:

6. The net adjustments – capital in the books of A and B


a. A, P10,000 net debit; B, P14,000 net credit
b. A, P10,000 net credit; B, P14,000 net debit
c. A, P14,000 net credit; B, P4,000 net debit
d. A, P14,000 net debit; B, P4,000 net credit

7. The adjusted capital of A and B in their respective books


a. A – P126,000; B – P214,000
b. A – P130,000; B – P204,000
c. A – P154,000; B – P196,000
d. A – P154,000; B – P186,000

8. The additional investment (withdrawal) made by A


a. P(30,000)
b. P6,000
c. P13,334
d. P16,667

9. The capital balances of A and B in the combined balance sheet


a. A, P124,000; B, P186,000
b. A, P162,500; B, 150,000
c. A, P162,500; B, 144,000
d. A, P200,000; B, 150,000
Use the following information for the next two (2) questions:

Pinnacle Corporation retails merchandise through its home office store and through a branch. Separate
ledgers are maintained by the home office and the branch. The branch purchases merchandise from
the home office (at 120% of home office cost), as well as from outside suppliers. Selected information
from the December 31 trial balances of the home office and branch is as follows:

Home Office Branch


Sales P375,530 P185,000
Shipments to branch 47,500 -
Purchases 453,560 28,000
Inventory, January 1 43,320 60,000
Shipments from home office - ?
Expenses 56,000 14,500
Branch inventory allowance 19,500 -

Additional information:
• The entire difference between the shipment account is due to the practice of billing and the
branch at cost plus 20%.
• The December 31 inventories are P80,000 and P56,000 for the home office and the branch
respectively. (The branch purchased 25% of its ending inventory from outside suppliers).
• Pinnacle determined that none of the shipments from home office was in-transit by the end of
the year.

Determine the following:

10. Overvaluation of Cost of Goods Sold


a. P7,000
b. P12,500
c. P19,500
d. P22,000

11. Adjusted branch net income


a. P72,000
b. P81,500
c. P87,000
d. P94,000

Use the following information for the next two (2) questions:

A, B and C formed a partnership on January 1 and had the following initial investment:

A P500,000
B 750,000
C 1,125,000

The partnership agreement states that the profits and losses are to be shared equally by the
partnership after consideration is made for the following:

• Salaries allowed to partners: P300,000 for A, P240,000 for B, and P180,000 for C.
• Average partner’s capital balances during the year shall be allowed 10% interest.
• On June 30, A invested additional P300,000.
• C withdrew P350,000 from the partnership on September 30.
• Share on the remaining partnership profit was P25,000 for each partner.

12. What is the total interest on the average capital balances of the partners?
a. P243,750
b. P268,750
c. P288,125
d. P303,125
13. What is the partnership net profit on December 31 before salaries, interests and partner’s share
on the remainder?
a. P998,750
b. P1,038,750
c. P1,058,125
d. P1,113,750

14. On April 30, A, B, and C form a partnership by combining their separate business proprietorships.
A contributed cash of P50,000. B contributed property with a P36,000 carrying amount, a P40,000
original cost, and P80,000 fair value. The partnership accepted responsibility for the P35,000
mortgage attached to the property. C contributed equipment with a P30,000 carrying amount, a
P75,000 original cost, and P55,000 fair value. The partnership agreement specifies that profits and
losses are to be shared equally but is silent regarding capital contributions. Which partner has the
largest capital account balances as of April 30?
a. A
b. B
c. C
d. All capital account balances are equal

15. Which of the following statements is true concerning the distribution of safe payments?
a. Safe payments are equal to the recorded capital balances of partners with positive capital
balances
b. The distribution of safe payments assumes that any capital deficit balances will prove to be a
total loss to the partnership
c. The distribution of safe payments may only be made after all liabilities have been paid
d. In computing safe payments, partners with positive capital balances are assumed to absorb
an equal share of any deficit balance(s)

16. The main difference between the net income reported in the separate income statement of the
branch and the net income reported by the home office for the branch’s operation is the
a. Overstatement of total goods available for sale reported by the branch
b. Overstatement of costs of goods sold reported by the branch
c. Overstatement of shipment from home office reported by the branch
d. Overstatement of beginning and ending inventory reported by the branch

17. What is the preferred method of resolving a partner’s deficit balance?


a. The partner with a deficit balance must contribute personal assets to cover the deficit,
regardless of personal condition
b. The partner with a deficit balance must contribute personal assets to cover the deficit only if
the partner’s personal assets exceed personal liabilities
c. The other partners must contribute personal assets to cover the deficit balance
d. The partnership mut sell assets in order to cover the deficit balance
Use the following information for the next three (3) questions:

D, E and F are partners sharing profits and losses of 50%, 30% and 20%, respectively. The December
31 balance sheet of the partnership before any profit allocation was summarized as follows:

Assets Liabilities and Capital


Cash P120,000 Accounts payable P8,000
Inventories 80,000 F, loan 6,000
Furniture and fixtures 100,000 D, capital 140,000
Patent 30,000 E, capital 120,000
F, capital 60,000
F, drawings (4,000)
Total assets P330,000 Total liabilities and capital P330,000

The partnership net income for the year amounted to P60,000

The next day, on January 1, F has decided to retire from the partnership and by mutual agreement
among partners, the following have been arrived at:
• Inventories amounting to P10,000 is considered obsolete and must be written-off.
• Furniture and fixtures should be adjusted to their current value of P130,000.
• Patents are considered worthless and must be written-off immediately before the retirement of F.

It was agreed that the partnership will pay F for his interest in the partnership inclusive of loan balance.

18. The interest of F immediately before his retirement amounted to


a. P48,000
b. P70,000
c. P72,000
d. P74,000

19. If F retires by receiving P76,000 cash, using bonus method, the capital balances of D and E after
the retirement of F
a. D, P162,500 and E, P133,500
b. D, P163,750 and E, P134,250
c. D, P165,000 and E, P135,000
d. D, P167,500 and E, P136,500

20. If F retires by receiving P69,000 cash, using bonus method, the capital balances of D and E after
the retirement of F
a. D, P165,625 and E, P135,375
b. D, P165,000 and E, P135,000
c. D, P166,875 and E, P136,125
d. D, P168,125 and E, P136,875

Use the following information for the next two (2) questions:

ABC Corporation operates a branch in Cebu City. At the close of business on December 31, Cebu Branch
account in the home office books showed a debit balance of P225,770. The interoffice accounts were
in agreement at the beginning of the year. For purposes of reconciling the interoffice accounts, the
following facts were ascertained:

I. An office equipment costing the home office P3,500 was picked up by the branch as P350.
II. Insurance premium of P675 charged by the home office was taken up twice by the branch.
III. Freight charges on merchandise made by the home office for P1,125 were recorded in the branch
book as P1,215.
IV. Home office credit memo representing a discount on merchandise for P800 was not recorded by
the branch.
V. The branch failed to take up a P700 debit memo from the home office representing the share of
the branch in the advertising.
VI. The home office inadvertently recorded a remittance for P3,000 from the Davao branch as a
remittance from its Cebu branch.
21. What is the balance of the Home Office account before adjustment as of December 31?
a. P225,000
b. P225,770
c. P226,485
d. P228,485

22. What is the adjusted balance of the Home Office account as of December 31?
a. P225,000
b. P225,770
c. P226,485
d. P228,770

Use the following information for the next two (2) questions:

ABC Company uses the weighted-average method in its process costing system. Data concerning the first
processing department for the most recent month are listed below:

Beginning work in process inventory:


Units in beginning work in process inventory .............................. 500
Materials costs .............................................................................. P8,500
Conversion costs ........................................................................... P14,700
Percent complete with respect to materials ................................ 85%
Percent complete with respect to conversion.............................. 55%

Units started into production during the month ............................. 5,300


Units transferred to the next department during the month.......... 4,400
Materials costs added during the month ......................................... P79,100
Conversion costs added during the month ...................................... P210,700

Ending work in process inventory:


Units in ending work in process inventory ................................... 1,400
Percent complete with respect to materials ................................ 50%
Percent complete with respect to conversion.............................. 30%

23. The cost per equivalent unit for materials for the month in the first processing department is
closest to:
a. P13.64
b. P15.51
c. P15.10
d. P17.18

24. The cost per equivalent unit for conversion costs for the first department for the month is closest
to:
a. P38.86
b. P43.71
c. P46.76
d. P49.10
Use the following information for the next two (2) questions:

The capital balances of partners Q, R, S are the following before liquidation: P87,000, P95,500,
P106,250 respectively. The partnership has a loan from partner Q in the amount of P8,000; loan to
partner R in the amount of P4,500; advances to partner S in the amount of P6,500. The partners’ profit
and loss ratio is 25:40:35 respectively.

25. If in the first installment the total cash paid to partners is P57,000, how much did partner S receive?
a. P0
b. P13,854
c. P19,396
d. P20,125

26. If partner Q received the full priority amount in the first instalment and partner S received P12,396
in the second installment, how much is received by partner Q as of the second instalment?
a. P8,854
b. P12,604
c. P23,750
d. P32,604

Use the following information for the next three (3) questions:

The following journal entries without peso data were taken from the accounting records of a company that
has a job-order costing system in which overhead is applied to jobs using a predetermined overhead rate.

1. Work in Process XX
Manufacturing Overhead XX
Wages Payable XX

2. Salary Expense XX
Wages Payable XX

3. Manufacturing Overhead XX
Accumulated Depreciation XX

4. Work in Process XX
Raw Materials XX

5. Work in Process XX
Manufacturing Overhead XX

6. Manufacturing Overhead XX
Raw Materials XX

7. Finished Goods XX
Work in Process XX

8. Raw Materials XX
Accounts Payable XX

27. The entry to transfer the cost of goods manufactured (completed goods) for the period is:
a. 1
b. 4
c. 5
d. 7

28. The entry to record the application of overhead is:


a. 1
b. 3
c. 5
d. 6
29. The entry to record depreciation on manufacturing equipment is:
a. 1
b. 3
c. 4
d. 5

Use the following information for the next two (2) questions:

Partners A, B, C, and D have been operating ABCD Partnership for ten years. Due to a significant
reduction in the demand for their product over recent years, the partners have agreed to liquidate the
partnership. At the time of liquidation, balance sheet accounts consisted of
• Cash, P103,500
• Noncash assets, P300,000
• Liabilities to outsiders, P60,000
• Capital credit balances for partners A, B, and C, P90,000, P150,000, and P120,000, respectively;
and a debit capital balance for partner D of P16,500.

Partners share equally in income and loss. It is estimated that the administrative cost of liquidation will
total P4,500. While preparing for liquidation, an unrecorded liability of P7,500 was discovered.

30. For how much must the noncash assets be sold for partner D to receive at least P5,000?
a. P386,000
b. P398,000
c. P429,500
d. P501,500

31. Assuming the available cash of P103,500 was distributed, how much must be the share of partner
B?
a. P0
b. P30,750
c. P31,500
d. P65,167

Use the following information for the next two (2) questions:

The A, B, and C Partnership is being liquidated. All liabilities have been paid. The balance of assets on
hand is being realized gradually. The following are details of partners’ accounts:

Capital account Drawing account Loans to P/L


balances balances partnership ratio
A P200,000 P15,000 Cr. P150,000 50%
B 250,000 20,000 Dr. - 20%
C 100,000 30,000 Cr. 50,000 30%

32. If you are to rank the partners from the most vulnerable to the least vulnerable, the ranking will
be as follows
a. A, B, and C, respectively
b. B, A, and C, respectively
c. C, B and A, respectively
d. C, A and B, respectively

33. If partner A receives P150,000, how much partner B receives?


a. P51,000
b. P86,000
c. P129,000
d. P144,000

34. Liquidation of the partnership refers to the winding up of the partnership’s affairs and conversion
of non-cash assets of partnership to cash with corresponding distribution to the appropriate party.
In the liquidation of the partnership, which of the following shall be paid last?
a. Due to partners other than capital and profit
b. Due to partners for his capital and profit
c. Due to third persons
d. Liquidation expenses
35. Which of the following statements concerning the presentation and measurement principles of
accounts regarding home office-branch accounting is incorrect?
a. The investment in branch account shall be reported as asset by the home office in its separate
statement of financial position
b. The home office current account shall be reported as equity by the branch in its separate
statement of financial position
c. The branch records inventories coming from the home office at cost while those coming from
outside supplier at billed price in its separate financial statement
d. The home office recognizes and measures all its inventories including those of the branch
initially at historical cost

Use the following information for the next three (3) questions:

Pinnacle Company had only one job in process on May 1. The job had been charged with P1,800 of direct
materials, P6,966 of direct labor, and P9,936 of manufacturing overhead cost. The company assigns
overhead cost to jobs using the predetermined overhead rate of P18.40 per direct labor-hour. During May,
the activity was recorded:

Raw materials (all direct materials):


Beginning balance ..................................................... P8,500
Purchased during the month .................................... P38,000
Used in production.................................................... P39,300
Labor:
Direct labor-hours worked during the month .............. 1,900
Direct labor cost incurred ............................................. P24,510
Actual manufacturing overhead costs incurred ........... P33,300
Inventories:
Raw materials, May 30.............................................. ?
Work in process, May 30 .......................................... P16,937

Work in process inventory on May 30 contains P3,741 of direct labor cost. Raw materials consist solely of
items that are classified as direct materials.

36. The balance in the raw materials inventory account on May 30 was
a. P1,300
b. P7,200
c. P29,500
d. P30,800

37. The cost of goods manufactured for May was


a. P97,110
b. P98,770
c. P100,535
d. P110,600

38. Assuming the difference is insignificant, the entry to dispose of the underapplied or overapplied
overhead cost for the month would include a:
a. Debit of P5,336 to Manufacturing Overhead
b. Debit of P1,660 to Cost of Goods Sold
c. Credit of P5,336 to Manufacturing Overhead
d. Credit of P1,660 to Cost of Goods Sold

39. The home office will credit the Branch account when
a. It takes up branch profits
b. It allocates expenses to the branch that were paid by the home office
c. Shipments of merchandise are made to the branch
d. It records the receipts of cash from the branch
40. X, Y, and Z are partners with capital balances of P50,000, P30,000, P20,000, respectively. The
partners share income and loss equally. For an investment of P50,000 cash, W is to be admitted as
a partner with a one-fourth interest in capital and income. Based on this information, the amount
of W’s investment can best be justified by which of the following?
a. W will receive a bonus from the other partners upon her admission to the partnership
b. The books value of the partnership’s net assets was less than their fair value immediately
prior to W’s investment
c. W is apparently bringing goodwill into the partnership, and her capital account will be
credited for the appropriate amount
d. Assets if the partnership were overvalued immediately prior to W’s investment

41. The following information were taken from the statement of realization and liquidation of Pinnacle
Company in Receivership for the month ended December 31:

Assets: Liabilities:
Assets to be realized P1,900,000 Liabilities to be liquidated P1,300,000
Assets acquired 100,000 Liabilities assumed 30,000
Assets realized 600,000 Liabilities liquidated 700,000
Assets not realized 840,000 Liabilities not liquidated 637,000

Profits and Loss:


Supplementary charges P180,000 Supplementary credits P603,000

How much is the profit or loss for the month of December?


a. P144,000 profit
b. P144,000 loss
c. P423,000 profit
d. P423,000 loss

Use the following information for the next three (3) questions:

ABC Corporation is currently undergoing liquidation due to financial difficulties. The trustee of ABC
presented the following information:

a. Assets amounting to P125,000 are available to unsecured liabilities without priority.


b. Assets amounting to P110,000 represents assets originally not pledged to any liabilities.
c. Unpaid liabilities are as follows: administrative expenses: P21,000; taxes: P18,000 and wages:
P32,000.
d. Accounts payable and notes payable totaled P180,000. No assets were pledged on the said
liabilities.
e. Payment to fully secured creditors and partially secured creditors amounts to P139,000 and
P144,000 respectively.
f. The expected recovery percentage is 40%.

42. Total liabilities


a. P522,500
b. P534,000
c. P580,000
d. P613,500

43. Amount of assets pledged to fully secured creditors


a. P139,000
b. P225,000
c. P235,000
d. P249,000

44. Total payment to all creditors


a. P187,500
b. P210,000
c. P426,000
d. P479,000
Use the following information for the next three (3) questions:

The following balances were ascertained in Mr. Accounting Company which is experiencing insolvency:
(round recovery percentage to two decimal places).

Cash P8,000 Accounts payable P80,000


Notes receivable 120,000 Accrued expenses 30,000
Inventories 80,000 Salaries payable 15,000
Prepaid expenses 10,000 Mortgage payable 155,000
Equipment, net 150,000 Ordinary shares 100,000
_______ Deficit (12,000)
Total P368,000 Total P368,000

Additional information:
a. Estimated net realizable value of the notes receivable was P105,000 and was pledged to
the mortgage payable.
b. 80% of the book value of the inventories can be sold at P45,000 and was pledged to 60%
of the accounts payable.
c. The remaining book value of the inventories have an estimated fair value of P20,000.
d. 80% of the remaining unpaid accounts payable were secured by the equipment having an
estimated fair value of P60,000.
e. Liquidation and administration expenses were estimated in the amount of P8,000.
f. Income tax payable had been accrued in the amount of P2,000.
g. Interest on the notes receivable and mortgage payable have not been accrued in the
amount of P10,000 and P15,000, respectively.

45. How much are the net free assets?


a. P52,400
b. P37,400
c. P31,000
d. P46,000

46. How much is the estimated payment to the mortgage payable?


a. P137,264
b. P146,191
c. P133,453
d. P142,379

47. How much is the estimated total recovery percentage of the partially secured accounts payable?
a. P97.29%
b. P95.85%
c. P96.28%
d. P96.86%
48. CPA Company operates several branches nationwide. On December 31, its Manila Branch showed
a home office account balance of P54,700. The following data are provided by the accountants of
the company:
• A P24,000 shipment, charged by Home Office to Manila Branch, was actually sent to and
retained by Makati Branch.
• A P30,000 shipment, intended and charged to Bulacan Branch was shipped to Manila Branch
and retained by the latter.
• A P4,000 emergency cash transfer from Makati Branch was not taken up in the Home Office
Books.
• Home Office collected a Manila Branch accounts receivable of P7,200 and failed to notify the
branch.
• Home Office was charged for P2,400 for merchandise returned by Manila Branch on
December 30. The merchandise is in transit.
• Home Office erroneously recorded Manila’s Branch net income for the year at P32,550. The
branch reported a net income of P25,350.

What is the unadjusted balance of the investment in Manila Branch account in the Home Office’s
book on December 31?
a. P40,300
b. P43,500
c. P51,100
d. P54,700

49. Which of the following transactions will decrease the investment in branch’s account in the home
office’s separate statement of financial position?
a. Net income of the branch
b. Return by branch to home office of merchandise shipped
c. Credit memo received from the branch
d. Payment of branch’s liability by the home office

50. Which of the following transactions will not affect the total equity of a partnership?
a. Withdrawal by a partner
b. Recognition of loss in case of admission of a new partner by investment
c. Retirement of an existing partner with payment of above the book value of such interest
d. Admission of a new partner by purchase of existing partner’s interest below its book value

51. Unacceptable units of production that are subsequently repaired and sold as good units are:
a. Scrap
b. Waste
c. Spoilage
d. Reworked units

52. CPA Company manufactures products A, B, and C from a joint process. Additional information is as
follows:
A B C Total
Units sold 2,000 1,000 500 3,500
Units on hand 2,000 1,000 500 3,500
Sales value at split-off ? ? P15,000 P100,000
Joint cost P36,000 ? ? P60,000
Cost after split-off P7,000 P5,000 P3,000 ?
Sales value at final point P70,000 P30,000 P20,000

Assuming the joint cost are allocated using the relative sales value method, what joint costs were
allocated to product B and what is the sales value at split-off of A, respectively:
a. P12,000 and P60,000
b. P14,286 and P63,000
c. P15,000 and P60,000
d. P16,000 and P63,000
53. ABC makes two products: Y and Z. They are initially processed from the same materials and then
after split-off, further processed separately. Additional information is as follows:

Y Z Total
Final sales value P45,000 P35,000 P80,000
Sales value at split-off 32,000 28,000 60,000
Cost beyond split-off 5,000 6,000 11,000
Joint cost prior to split-off 18,000

Using the Approximated Net Realizable Value approach, how much is the joint cost assigned to Y
and Z?
a. P7,500 and P7,500
b. P9,600 and P8,400
c. P9,918 and P8,082
d. P10,435 and P7,565

54. Prior to partnership liquidation, a safe payments schedule is frequently prepared to determine the
amount of cash that may be safely distributed to the partners. The safe payment schedule
a. Indicates the distribution of successive amounts of available cash to each partner
b. Consists of each partner’s capital account plus loan balance, divided by that partner’s profit-
and-loss sharing ratio
c. Assumes contribution of personal assets by partners unless there is a substantial
presumption of personal insolvency by the partners
d. Shows the successive losses necessary to eliminate the capital accounts of partners assuming
no contribution of personal assets by partners

55. If the spoilage in process costing is considered to be continuous, which of the following statements
is incorrect?
a. The cost assigned to abnormal loss shall be treated as period cost or expense
b. The cost assigned to normal loss shall be allocated to units completed, work-in-process
ending inventory and abnormal loss
c. The normal loss shall be given 100% equivalent unit of production while abnormal loss shall
not be given an equivalent unit of production to automatically allocate the cost of normal
loss to units completed, work-in-process ending inventory and abnormal loss by increasing
the cost per unit
d. None of the above

56. BSA Company provided the following information for transactions that occurred during October.
The corporation uses Just-in-Time costing system.
• Raw materials purchased and requisitioned for product were P110,000.
• Direct Labor costs of P130,000 were incurred.
• Actual factory overhead costs amounted to P125,000.
• Applied conversion costs totaled P270,000. This included P130,000 of direct labor.
• All units were completed.

How much is the balance in Finished Goods account in October 31?


a. P365,000 debit
b. P380,000 debit
c. P365,000 credit
d. P380,000 credit
Use the following information for the next two (2) questions:

RC Corp. had provided the following information for the transactions that occurred during September. The
company uses the JIT costing system:
• Raw materials costing P750,000 were purchased.
• All materials costing P550,000 were used in production.
• Direct Labor costs of P600,000 were incurred.
• Actual factory overhead costs amounted to P1,250,000.
• Applied conversion costs totaled P2,300,000. This includes the direct labor cost.
• All units are completed and immediately sold.

57. Determine the unadjusted cost of goods sold for the month of September:
a. P2,400,000
b. P2,600,000
c. P2,850,000
d. P3,450,000

58. Using the preceding information, what is the over or under applied conversion costs?
a. P300,000 under
b. P450,000 over
c. P450,000 under
d. P1,050,000 over

59. If there was no beginning work in process and no ending work in process under the weighted-
average process costing method, the number of equivalent units for direct materials, if direct
materials were added at the start of the process, would be:
a. Equal to the units started or transferred in
b. Equal to the units completed
c. Less than the units completed
d. Both a and b are correct

60. The main difference between the FIFO and weighted average methods of process costing is
a. In the treatment of current period production costs
b. In the treatment of lost units
c. In the treatment of beginning Work in Process Inventory
d. In the treatment of ending Work in Process Inventory

61. Under the weighted-average method, the stage of completion of beginning work in process:
a. Is relevant in determining the equivalent units
b. Is irrelevant in determining the equivalent-unit calculation
c. Can almost always be determined with a high degree of precision
d. Must be combined with the work done during the current period to determine the equivalent
units

62. The following data were ascertained for the month of November in the Statement of realization
and liquidation of Mr. Accounting Corp.:
• Estate equity at the end of October was (18,500).
• Liabilities not paid at the end of October were 310,000.
• Liabilities assumed were 15,800.
• Assets sold and collected during the month were 235,000.
• Increase in assets were 36,500.
• Assets to be realized in December were 13,500.
• Liabilities to be paid in December were 30,800.
• Supplementary credits and charges were 100,800 and 28,750, respectively.
• Estate equity at the end of the month was (12,450).

What is the beginning cash balance for the month of November?


a. 4,850
b. 8,500
c. 2,000
d. 13,500
63. Transferred-in costs are treated as if they are:
a. Direct labor costs added at the beginning of the process
b. A separate direct material added at the beginning of the process
c. Conversion costs added at the beginning of the process
d. Costs of beginning inventory added at the beginning of the process

64. ABC Corp. manufactures a product that yields a by-product “X”. The only costs associated with X
are selling costs of P0.10 for each unit sold. ABC Corp. accounts for sales X by deducting X’s
separable costs from X’s sales, and then deducting this net amount from the major product’s cost
of goods sold. X’s sales were 100 000 units at P1.00 each. If ABC Corp. changes its method of
accounting for X’s sales by showing the net amount as additional sales revenue, then ABC Corp’s
gross margin would be?
a. Be unaffected
b. Decreased to P90,000
c. Increased to P110,000
d. Increased to P100,000

65. In an advance plan for installment distributions of cash to partners of a liquidating partnership,
each partner’s loss absorption potential is computed by
a. Multiplying each partner’s capital account balance by the percentage of that partner’s capital
account balance to partner’s capital
b. Dividing the total of each partner’s capital account less receivables from the partner plus
payables to the partner by the partner’s profit and loss percentage
c. Dividing each partner’s capital account balance by the percentage of that partner’s capital
account balance to partnership’s total capital
d. None of the above
Use the following information for the next five (5) questions:

ABC Company manufactures a single product being used for heavy industrial machineries. It operates on
three shifts in two departments – Cutting Department and Smoothing Department. Materials are added
to the product at the start of each department. The following records for each department were
obtained from the books of ABC Company for the month of January:

Cutting Smoothing
Units in process, January 1 0 0
Units started in production 100, 000 -
Units completed and transferred out 80, 000 60,000
Units in process, January 31 20, 000 ?
Units spoiled in production (abnormal) 0 2, 000

Spoiled units are 50% complete as to materials, labor and overhead. Percentage of completion of units
in process on January 31 follows:
Cutting Smoothing
Materials 100% 100%
Labor 60% 80%
Overhead 20% 40%

The following charges were indicated in the cost records for the month of January:
Cutting Smoothing
Materials P300, 000 P118, 500
Labor 184,000 203, 580
Overhead 100,800 75,020
66. The total cost transferred to next department - Smoothing
a. P496,000
b. P556,000
c. P584,800
d. P690,000

67. The cost of work-in-process ending in Cutting Department


a. P4,800
b. P60,000
c. P84,000
d. P88, 800

68. The total cost transferred to finished goods


a. P496,000
b. P556,000
c. P584,800
d. P690,000

69. The total cost transferred to spoiled units


a. P0
b. P6,200
c. P11,500
d. P17,700

70. The cost of work-in-process ending in Smoothing Department


a. P11,600
b. P73,800
c. P138,600
d. P185,400

- - - END - - -

You might also like